You are on page 1of 122

1M


=: ltem_1of30
? 61 � � , � I)
2 Question Id: 7504
:.
- ?Mark <J
Previous Next
[:> Tutorial Lab Values Notes Calculator Reverse Color Text Zoom
3
4 A 44-year-old woman requests a same-day appointment due to poor sleep. She says that when she goes to bed,
5
her "brain can't shut off' and she feels "too wired" to fall asleep. The patient is well known to the physician and has
6
7
a long history of bipolar disorder. Her medications include valproate and ziprasidone. Physical examination is
8 normal. On mental status examination, the patient is very animated, laughs frequently, and speaks very rapidly.
She has no hallucinations, and no delusions are elicited. The physician increases the patient's valproate dosage
� and prescribes a 1-week supply of zolpidem for sleep with a planned follow-up in a week. At the close of the visit,
11
the patient thanks the physician profusely and presents him with a gift of a bottle of wine, saying she is so
12
appreciative that he was willing to see her on such short notice. Which of the following is the most appropriate
13
14 response by the physician?
15
16 O A. I appreciate the gift. Thank you very much.
17
18
O B. I appreciate your thanks, but I am unable to accept the gift.
19 O C. Thank you, but I can accept this gift only if it cost less than $100.
20
21 O D. Thank you, but physicians are prohibited from accepting any gifts.
22
23
O E. Thank you for the gift, but please understand that it does not affect the care you receive.
24
25
Submit
26
27

Block Time Remaining: 00:44:48


28
29 TIMEDTUTOR

03:38�
T• l/vl l/1 •
1M

=: ltem_1of30
? 61 � � , � I)
2 Question Id: 7504
:.
- ?Mark <J
Previous Next
[:> Tutorial Lab Values Notes Calculator Reverse Color Text Zoom
3 •
Accepting gifts from patients often raises complex issues. Although some physicians may decide to adopt a no-gift
4
5
policy, others maintain that gifts of low value that are not given with the expectation of preferential treatment are
6 acceptable, and that rejecting the gift can damage the physician-patient relationship. In the absence of absolute
7 guidelines, most physicians make decisions on a case-by-case basis and consider multiple factors, including the
8 cost, type, and timing of the gift; the apparent motivation behind giving it; and the nature and longevity of the
physician-patient relationship. Gifts that are clearly extravagant, inappropriately intimate, or given to secure

11
preferential treatment should not be accepted.
12 In this case, the primary concern is the patient's unstable mental state. She has a known diagnosis of bipolar
13
disorder and her report of difficulty sleeping, racing thoughts, and elevated mood and pressured speech on mental
14
15
status examination are consistent wi th a hypomanic episode. The physician must consider that this patient's gift
16 giving is a manifestation of impaired judgment associated with her psychiatric cond ition. The gift should be politely
17 declined.
18
19 (Choice A) It would be inappropriate for the physician to accept a gift from this unstable patient.
20
(Choice C) There is no definite value that determines the appropriateness or inappropriateness of a gift. The
21
main issue in this case is the mental status of the patient, not the value of the g ift.
22
23 (Choice D) There is no absolute prohibition on accepting gifts, and it would be inaccurate to tell the patient this.
24
However, if the physician has a personal rule not to accept gifts, this could be communicated to the patient.
25
26 (Choice E) Although physicians should ensure (and communicate) that accepting gifts wi ll not alter the patient's
27 level of care, it is not appropri ate to accept this gift due to the patient's condition. •
28
Block 1...... • ,""'
............, .....
v _.....·45
29 TIMEDTUTOR
1M

=: ltem_1of30
? 61 � � , � I)
2 Question Id: 7504
:.
- ?Mark <J
Previous Next
[:> Tutorial Lab Values Notes Calculator Reverse Color Text Zoom
3 In this case, the primary concern is the patient's unstable mental state. She has a known diagnosis of bipolar •
4
disorder and her report of difficulty sleeping, racing thoughts, and elevated mood and pressured speech on mental
5
status examination are consistent wi th a hypomanic episode. The physician must consider that this patient's gift
6
7 giving is a manifestation of impaired judgment associated with her psychiatric condition. The gift should be politely
8 declined.

(Choice A) It would be inappropriate for the physician to accept a gift from this unstable patient.

11 (Choice C) There is no definite value that determines the appropri ateness or inappropriateness of a gift. The
12
main issue in this case is the mental status of the patient, not the value of the g ift.
13
14 (Choice 0) There is no absolute prohibition on accepting gifts, and it would be inaccurate to tell the patient this.
15 However, if the physician has a personal rule not to accept gifts , this could be communicated to the patient.
16
17 (Choice E) Although physicians should ensure (and communicate) that accepting gifts wi ll not alter the patient's
18 level of care, it is not appropriate to accept this gift due to the patient's condition.
19
20
Educational objective:
21 Considerations in accepting gifts from patients include the cost, type, timing, and intention of the gift. Gifts given
22 by patients with an unstable mental illness that could be affecting their judgment should not be accepted.
23
24
References
25
26 • Accepting gifts from patients.
27

Block Time Remaining: 00:44:45


28
29 TIMEDTUTOR

03:38�
T• l/vl l/1 •
1M
1 •
=: ltem_2of30
? 61 � � , � I)
Question Id: 4846
:.
- ?Mark <J
Previous Next
[:> Tutorial Lab Values Notes Calculator Reverse Color Text Zoom
3 •
4 A 1-week-old girl is brought to the emergency department by ambulance due to fever and irritability. She was born
5
at term via precipitous vaginal delivery and had an uncomplicated nursery course. Prenatal laboratory results
6
7
included a positive group 8 streptococcal screen, for which the mother received intrapartum antibiotics. The
8 infant's mother and father are both age 16, and the father is not involved in the infant's care. The infant lives with
her mother and maternal grandparents in their home. The mother is present in the emergency department along
� with both of her parents. Temperature is 38.3 C (101 F) and pulse oximetry is 87% on room air. The infant is
11
intermittently apneic and appears cyanotic. Anterior and posterior fontanelles are bulging. Pulmonary examination
12
reveals tachypnea, intercostal retractions, and clear breath sounds bilaterally. The infant's examination is
13
14 concerning for meningitis and the child requires intubation and mechanical ventilation; however, the mother refuses
15 the procedure. Which of the following is the best next step in management of this patient?
16
17 O A. Attempt to contact infant's father for consent
18
19 0 8. Call local child protective services
20
21
O C. Contact hospital legal counsel for legal guardianship
22 O D. Obtain consent from infant's maternal grandparents
O E. Place urgent consult request to hospital ethics committee
23
24
25
26
O F. Proceed with mechanical ventilation without consent
27 •
Block Time Remaining: 00:44:43
28
29 TIMEDTUTOR

03:38�
T• l/vl l/1 •
1M
1 �
=: ltem_2of30
? 61 � � , � I)
Question Id: 4846
:.
- ?Mark <J Previous Next
[:> Tutorial
- -
Lab Values Notes Calculator Reverse Color Text Zoom
3 - - ' .. • - • el J.. ■ .. •- ,- ' .. • ... e ... I • • ... -• I - t el - ... ... ... ee ... • t - I

4 included a positive group 8 streptococcal screen, for which the mother received intrapartum antibiotics. The
5 infant's mother and father are both age 16, and the father is not involved in the infant's care. The infant lives with
6
her mother and maternal grandparents in their home. The mother is present in the emergency department along
7
8 with both of her parents. Temperature is 38.3 C (101 F) and pulse oximetry is 87% on room air. The infant is
intermittently apneic and appears cyanotic. Anterior and posterior fontanelles are bulging. Pulmonary examination

reveals tachypnea, intercostal retractions, and clear breath sounds bilaterally. The infant's examination is
11 concerning for meningitis and the child requires intubation and mechanical ventilation; however, the mother refuses
12
the procedure. Which of the following is the best next step in management of this patient?
13
14
X � A. Attempt to contact infant's father for consent [2%>]
15
16 8. Call local child protective services [1 %]
17
18 C. Contact hospital legal counsel for legal guardianship [1%]
19
D. Obtain consent from infant's maternal grandparents [2%]
20
21 E. Place urgent consult request to hospital ethics committee [2%]
22
23 F. Proceed with mechanical ventilation without consent [88%]
24
25
26
lnco�ct
27 (1,, 88% I I I:
L!!!. Answered correctl II • •I
Block Time Remaining: 00:44:36
28
29 TIMEDTUTOR

03:38�
T• l/vl l/1 •
1M
� � , �

ltem 2of30
1
=: _
Question Id: 4846
:.
- ?
Mark <J
Previous Next
[:>
?
Tutorial
61
Lab Values Notes Calculator Reverse Color Text Zoom
I)
3 •
4 Circumstances in which minors can provide their own consent
5
6 • Emergency care (all states)
7 • Sexually transmitted infection (all states)
8 Medical emancipation • Mental health and substance abuse treatment
• Pregnancy care
� • Contraception
11

12 • Financially independent
13
• Parent
14
Legal emancipation • Married
15
16 • Active military service
17 • High school graduate
18
19
Note: The age of legal and medical emancipation varies by state.
20
Informed consent provides patients with information that allows for educated decision-making about their health
21
22 care. The procedure and its indications, risks, benefits, and alternatives should be clearly described. Informed
23 consent ideally is obtained from the parents of minors prior to performing a procedure. The exception is
24 emergency, life-threatening conditions in which the parent or legal guardian cannot or does not provide consent
25
and delaying treatment would result in harm to the patient.
26
27 Although this infant's mother is a minor, she can legally consent to medical care for her child. However, parents •
Block Time Remaining: 00:44:36
28
29 TIMEDTUTOR

03:38�
T• l/vl l/1 •
1M
1 •
=: ltem_2of30
? 61 � � , � I)
Question Id: 4846
:.
- ?Mark <J
Previous Next
[:> Tutorial Lab Values Notes Calculator Reverse Color Text Zoom
3 •
4
Although this infant's mother is a minor, she can legally consent to medical care for her child. However, parents
5 have the responsibility to make decisions in the best interest of their child and therefore cannot refuse life­
6 saving treatment. This infant requires intubation and mechanical ventilation to live. In situations that are not
7 time-sensitive, the hospital legal counsel and ethics committee should be involved and a court order obtained to
8
overrule the parent's wishes (Choices C and E). However, this patient has an unstable airway requiring
emergency treatment; obtaining an ethics consult or legal counsel would delay treatment. The correct course of

11 action is to proceed with treatment regardless of parental consent. Other situations in which obtaining consent
12 should not delay acute care include urgent blood transfusions and emergency surgery.
13
14
(Choices A and D) Consent is not required in this emergency situation, and attempting to contact the father would
15 only delay life-saving treatment. In addition, grandparents cannot provide consent unless they are the legal
16 guardians.
17
18
(Choice B) Child protective services should be involved when parents are not acting in the best interest of their
19 child. However, this patient is in criti cal condition and should first be stabilized.
20
Educational objective:
21
22 Parents who are themselves minors can g ive consent to medical care for their child. However, consent is not
23 required in emergency situations, and parents cannot refuse life-saving treatment for their child.
24
25
References
26
27 • Consent for emergency medical services for children and adolescents. •
Block Time Remaining: 00:44:36
28
29 TIMEDTUTOR

03:38�
T• l/vl l/1 •
1M
1 •
=: ltem_ 3of30
? 61 � � , � I)
2 Question Id: 3742
:.
- ?Mark <J
Previous Next
[:> Tutorial Lab Values Notes Calculator Reverse Color Text Zoom

4 A 34-year-old obese woman is brought to the emergency department complaining of severe ri ght upper-quadrant
5
abdominal pain. She is lying on her left side because "my right shoulder and back hurt too." She is accompanied
6
7
by her husband, who attempts to provide emotional support. Following ,n ultrasound, the patient is diagnosed with
8 acute cholecystitis. She is admitted to the medical service, but has progressive symptoms despite conservative
treatment and a surgical consult is ordered. The decision is made to operate, and the patient is discharged after
� an uncomplicated cholecystectomy. At discharge, the patient's husband requests a copy of all medical records
11
associated with her stay. Which of the following is the most appropriate course of action?
12
13
Q A. Ask the patient's husband to put his request in writing
14
15 Q B. Ask the patient to explain the purpose of the request
16
17 Q C. Immediately make copies of the patient's record and give them to the patient's husband
18
Q D. Inform the husband that an attorney must write a letter requesting the records
19
20 Q E. Inform the husband that the patient must request a copy of her medical records
21
22 Q F. Instruct the medical records department to mail the patient her original records as soon as possible
23
Q G. Provide a copy of the medical team's notes and explain that surgical notes will need to be obtained
24
separately
25
26
27
Submit
Block Time Remaining: 00:44:33
28
29 TIMEDTUTOR

03:38�
T• l/vl l/1 •
1M
1 •
=: ltem_ 3of30
? 61 � � , � I)
2 Question Id: 3742
:.
- ?Mark <J
Previous Next
[:> Tutorial Lab Values Notes Calculator Reverse Color Text Zoom

4 A 34-year-old obese woman is brought to the emergency department complaining of severe ri ght upper-quadrant
5
abdominal pain. She is lying on her left side because "my right shoulder and back hurt too." She is accompanied
6
7
by her husband, who attempts to provide emotional support. Following an ultrasound, the patient is diagnosed with
8 acute cholecystitis. She is admitted to the medical service, but has progressive symptoms despite conservative
treatment and a surgical consult is ordered. The decision is made to operate, and the patient is discharged after
� an uncomplicated cholecystectomy. At discharge, the patient's husband requests a copy of all medical records
11
associated with her stay. Which of the following is the most apprq::priate course of action?
12
13
A. Ask the patient's husband to put his request in writing [0%)
14
15 B. Ask the patient to explain the purpose of the request [2%>]
16
17 X �- C. I mmediately make copies of the patient's record and give them to the patient's husband (1 %]
18
D. Inform the husband that an attorney must write a letter requesting the records [0%]
19
20 E. Inform the husband that the patient must request a copy of her medical records [92%]
21
22 F. Instruct the medical records department to mail the patient her original records as soon as possible [1 %)
23
G. Provide a copy of the medical team's notes and explain that surgical notes will need to be obtained
24
25
separately [0%)
26
27

Block Time Remaining: 00:44:30


28
29 TIMEDTUTOR

03:38�
T• l/vl l/1 •
1M
1 •
=: ltem_ 3of30
? 61 � � , � I)
2 Question Id: 3742
:.
- ?Mark <JPrevious Next
[:> Tutorial Lab Values Notes Calculator Reverse Color Text Zoom

4
The Health Insurance Portability and Accountability Act (HIPAA) states that, under federal law, patients have the
5 legal right to obtain a copy (not the original) of their medical records. Only the patient or a personal representative
6 has the right to access the record. Medical records cannot be released to a spouse, family member, or any other
7 person without the patient's authorization, except when directed by a written court order. Under HIPAA, a facility or
8
physician has 30 days to provide the patient or the patient's representative with a copy of the requested medical
records. Most hospitals process such requests through the medical records department and require the patient to

11 sign an authorization to release the records.
12
Health care providers are required to share all records, including progress notes, laboratory results, diagnostic
13
14
testing results, and billing information. They are also required to share another physician's notes (eg, in the case
15 of a consult or referral) if the note contains information that contributed to the diagnosis and treatment of the
16 patient. Any records received from another health care provider involved in the care and treatment of the patient
17 should be maintained as part of the patient's medical record.
18
19 (Choices A and C) The medical record request must come from the patient, not her spouse. In addition, there is
20 no requirement to provide medical records immediately on request. The timeframe is usually within 30 days of the
21
request.
22
23 (Choice B) Although it may be beneficial to explore the patient's reasons for requesting records, a patient is
24 legally entitled to inspect and receive a copy of the medical record without giving a reason.
25
26 (Choice D) An attorney' s letter is not needed for patients to request a copy of their medical records.
27
- . . .. . . of their records. not the ori inal records. Ph sicians are re
Block Time Remaining: 00:44:30
28
29 TIMEDTUTOR

03:38�
T• l/vl l/1 •
1M
1 •
=: ltem_ 3of30
? 61 � � , � I)
2 Question Id: 3742
. ... . .
:.
- ?Mark <J
Previous
- -- · - --- -
Next
[:>--
--- -- -- - - - -- - -
Tutorial
-
Lab Values
------ ---
Notes
- -
Calculator
- -
Reverse Color Text Zoom

4 no requirement to provide medical records immediately on request. The timeframe is usually within 30 days of the
5 request.
6
7
(Choice B) Although it may be beneficial to explore the patient's reasons for requesting records, a patient is
8 legally entitled to inspect and receive a copy of the medical record without giving a reason.

(Choice 0) An attorney' s letter is not needed for patients to request a copy of their medical records.

11 (Choice F) Patients are entitl ed to a copy of their records, not the original records. Physicians are required to
12 maintain the original records to ensure that a patient's medical history is available to assist future health care
13
providers in the patient's care.
14
15 (Choice G) A physician is required to share another physician's notes if they contain information that contributed
16 to the diagnosis and treatment of the patient. In this case, it would be appropriate to include the surgical notes.
17
18 Educational objective:
19 Under the Health Insurance Portability and Accountability Act, patients have the legal ri ght to obtain copies of their
20
medical records within a specified timeframe.
21
22
23 References
24
• The effects of promoting patient access to medical records: a review.
25
26 Copyright© UWo�d. All rigllts reserved.
27 •
Block Time Remaining: 00:44:30
28
29 TIMEDTUTOR

03:38�
T• l/vl l/l •
1M
1 •
=: ltem4of30
? 61 � � , � I)
2 Question Id: 3388 •
- ?
Mark <J
Previous Next
[:> Tutorial Lab Values Notes Calculator Reverse Color Text Zoom
3 A 68-year-old man is brought to the hospital due to 2 days of progressive shortness of breath and cough. He has a •
history of severe chronic obstructive pulmonary disease and has had multiple recent exacerbations and several
5
hosp italizations over the past year. During his last hospitalization, he had acute respiratory failure requiring
6
7 endotracheal intubation. This prompted the patient to make a living will specifying that he does not want any
8 resuscitative measures or invasive therapies in the event of a life-threatening emergency. On examination, he is
lethargic and arousable only to noxious physical stimuli. His temperature is 37.2 C (99 F), blood pressure is
� 132/70 mm Hg, pulse is 102/min, and respirations are 32/min. Examination reveals diffusely decreased breath
11
sounds, bilateral expiratory wheezes, and labored breathing. It is determined that he lacks decision-making
12
13 capacity. His daughter says, "I have been taking care of my father for many years and I know him better than
14 anyone. He would not want any aggressive measures. Please just make him comfortable." However, the patient's
15 wife demands that everything be done to save her husband's life, including intubation if necessary. She insists that
16
she knows what her husband would have wanted and threatens to file a lawsuit. Which of the following
17
interventi ons is the most appropriate next step?
18
19
20
O A. Consult the hospital ethics committee
21 O B. Meet with all family members to achieve consensus on a course of action
22
23 O C. Obtain a court order to proceed with intubation
24
25
O D. Proceed with endotracheal intubation if medically indicated
26 O E. Respect the patient's living will and provide comfort care only
27

Block Time Remaining: 00:44:26


28
29 TIMEDTUTOR

03:39 �
T• l/vl l/1 •
1M
1 •
=: ltem4of30
? 61 � � , � I)
2 Question Id: 3388 •
- ?
Mark <J
Previous Next
[:> Tutorial Lab Values Notes Calculator Reverse Color Text Zoom
3 •
endotracheal intubation. This prompted the patient to make a living will specifying that he does not want any
resuscitative measures or invasive therapies in the event of a life-threatening emergency. On examination, he is
5
6 lethargic and arousable only to noxious physical stimuli. His temperature is 37.2 C (99 F), blood pressure is
7 132/70 mm Hg, pulse is 102/min, and respirations are 32/min. Examination reveals diffusely decreased breath
8 sounds, bilateral expiratory wheezes, and labored breathing. It is determined that he lacks decision-making
capacity. His daughter says, "I have been taking care of my father for many years and I know him better than
� anyone. He would not want any aggressive measures. Please just make him comfortable." However, the patient's
11
12 wife demands that everything be done to save her husband's life, including intubation if necessary. She insists that
13 she knows what her husband would have wanted and threatens to file a lawsuit. Which of the following
14 interventi ons is the most appropriate next step?
15
16 A. Consult the hospital ethics committee [0%]
17
18 B. Meet with all family members to achieve consensus on a course of action [4%>]
19
X@ C. Obtain a court order to proceed with intubation [0%]
20
21 D. Proceed with endotracheal intubation if medically indicated [2%>]
22
23 E. Respect the patient's living will and provide comfort care only [92%]
24
25
26 �
Incorrect
27 (1, , 92%
L!!!. Answered correctl II • •I
28
Block Time Remaining: 00:44:22
29 TIMEDTUTOR

03:39 �
T• l/vl l/1 •
1M
1 •
=: ltem4of30 ? 61 � � , � I)
2 Question Id: 3388 •
- ?
Mark <J
Previous Next
[:> Tutorial Lab Values Notes Calculator Reverse Color Text Zoom
3 •

5
A living will (advanced directive) specifies a patient's wishes for health care in advance of losing the ability to
6 communicate or of becoming incapable of making his or her own decisions. In this case, the patient is
7 incapacitated and his family members strongly disagree on the best course of action; both claim to have a better
8 understanding of what the patient would have wanted in the current situation. However, the physician's
responsibility is to the patient, and ethically the physician must adhere to the patient's wishes as outlined in the

11
living will. The living will protects the patient's autonomy and overrules the wishes of any family members.
12 (Choices A, B, and C) Initiating a family meeting would be the first step if the family members disagreed about the
13
course of action and there was no living wi ll and/or no designated health care proxy in place. Referral to the
14
15
hospital ethics committee or the courts for a judgment would become necessary if no consensus was achieved.
16 (Choice D) Proceeding with CPR or intubation would violate the patient's wishes as expressed in his living will.
17
18 Educational objective:
19 A living will communicates the patient' s own wishes if he or she becomes incapacitated, and it overrules the wishes
20
of the family.
21
22
23 References
24
• Efficacy of advance care planning: a systematic review and meta-analysis.
25
26 Copyright© UWo�d. All rigllts reserved.
27 •
Block Time Remaining: 00:44:22
28
29 TIMEDTUTOR

03:39 �
T• l/vl l/l •
1M
1 •
=: ltem_5of30
? 61 � � , � I)
2 Question Id: 4320
:.
- ?Mark <J
Previous Next
[:> Tutorial Lab Values Notes Calculator Reverse Color Text Zoom
3 •
4 A 10-year-old girl is brought to the office by her mother for evaluation of recent behavioral changes. The patient
has been sleeping poorly and recently started wetting her bed. Her grades in school have dropped significantly,
6
and she has been notably anxious and sad. The patient's parents are having financial difficulties and considering
7
8 divorce. The father drinks alcohol most evenings after coming home from work. The mother appears stressed by
her husband's drinking and is very concerned about her daughter. The physician has known the patient since she

was a toddler and found her to be cheerful, lively, and cooperative at her checkup 6 months ago. After the mother
11 leaves the room, the physician asks the patient about school and home, at which point she suddenly bursts into
12
tears. After providing support, which of the following is the best next step in management of this patient?
13
14
15
O A. Ask the patient about physical, emotional, or sexual abuse
16 O B. Encourage the mother to attend a support group for family members of alcoholics
17
18 0 C. Interview the mother alone
19
20
0 D. Reassure patient and mother that this is a normal stress response
21 0 E. Recommend cognitive-behavioral therapy
22
23 0 F. Recommend family therapy
24
25
26 Submit
27 •
Block Time Remaining: 00:44:21
28
29 TIMEDTUTOR

03:39 �
T• l/vl l/1 •
1M
1 •
=: ltem_5of30
? 61 � � , � I)
2 Question Id: 4320
:.
- ?Mark <J
Previous Next
[:> Tutorial Lab Values Notes Calculator Reverse Color Text Zoom
3 •
4 A 10-year-old girl is brought to the office by her mother for evaluation of recent behavioral changes. The patient
has been sleeping poorly and recently started wetting her bed. Her grades in school have dropped significantly,
6
7
and she has been notably anxious and sad. The patient's parents are having financial difficulties and considering
8 divorce. The father drinks alcohol most evenings after coming home from work. The mother appears stressed by
her husband's drinking and is very concerned about her daughter. The physician has known the patient since she
� was a toddler and found her to be cheerful, lively, and cooperative at her checkup 6 months ago. After the mother
11
leaves the room, the physician asks the patient about school and home, at which point she suddenly bursts into
12
tears. After providing support, which of the following is the best next step in management of this patient?
13
14
A. Ask the patient about physical, emotional, or sexual abuse [84%]
15
16
B. Encourage the mother to attend a support group for family members of alcoholics [2%]
17
18 C. Interview the mother alone [1 %]
19
20
D. Reassure patient and mother that this is a normal stress response [1°/o]
21
E. Recommend cognitive-behavioral therapy [1%]
22
23 X (= F. Recommend family therapy [8%]
24
25
26
Incorrect
27 :�..
.!I ,• :.1 u- ,-.
I I:II :
I • :.1
Block Time Remaining: 00:44:19
28
29 TIMEDTUTOR

03:39 �
T• l/vl l/1 •
1M
1 •
=: ltem_5of30
? 61 � � , � I)
2 Question Id: 4320
:.
- ?Mark <J
Previous Next
[:> Tutorial Lab Values Notes Calculator Reverse Color Text Zoom
3 •
4 Features of possible child abuse

Caregiver background
6
7 • Young or single parents
8 • Lower education levels
• Drug or alcohol abuse
� • Psychiatric conditions (depression, impulse control disorders)
11
• History of childhood abuse
12
13 Risk Home environment
14 factors • Unstable family situation (eg, divorce, conflict)
15 • Financial difficulties, job loss
16
• Lack of social support
17
• Domestic violence
18
19 Victims
20 • Physical, intellectual, or emotional disabilities
21 • Unplanned pregnancy/unwanted child
22
23 • Unexplained or implausible injuries
24 • Injuries in different stages of healing
25
Clinical presentation
• Malnutrition
26
• Sudden behavioral or scholastic changes
27 •
Block Time Remaining: 00:44:19
28
29 TIMEDTUTOR

03:39 �
T• l/vl l/1 •
1M
1
2

=:
ltem 5of30
_
Question Id: 4320
:.
- ?
Mark <J
Previous Next
[:>
?
Tutorial
61
Lab Values

Notes

Calculator
,
Reverse Color

Text Zoom
I)
3 When working with vulnerable patients such as c hildren, it is imperative that physicians be alert to any signs of •
4
possible abuse. Abuse must be ruled out in any child wi th abrupt onset of mood c hanges, bedwetting, and/or

6
academic difficulties. Other factors associated with increased risk of abuse in this child include unstable family
7 situation, financial difficulties, and caregiver history of substance abuse.
8
Physical, sexual and emotional abuse should be considered early when evaluating mood and behavioral
symptoms in children due to the high risk of recurrent abuse and the need for prompt intervention. The physician

11
should ask the parent(s) to leave the room and provide a safe environment for the child to talk about her feelings
12 and concerns. If abuse is confirmed or there remains a high index of suspicion of abuse, the physician is
13 mandated to report it.
14
15 (Choices B and F) Although encouraging the mother to attend a support group and initiating family therapy may
16 be helpful, these would not take priority over assessing the patient for abuse.
17
18
(Choice C) It is important to speak with the mother alone to get further details about the home environment, which
19 she may not wish to reveal in front of her daughter . However, this does not take precedence over completing the
20 interview with the patient and assessing for abuse.
21
22
(Choice D) Changes in behavior and mood often occur in normal children in response to common psychosocial
23 stressors such as divorce or parental discord. However, abuse must also be ruled out when a child has abrupt
24 changes in mood, bedwetting, and academic decline.
25
26
(Choice E) Cognitive-behavioral therapy has been studied for a number of psychological disorders in children and
27 adolescents. Al though it may have a role in the long-term management of this patient, psychotherapy would be a •
Block Time Remaining: 00:44:19
28
29 TIMEDTUTOR

03:39 �
T• l/vl l/1 •
1M
1 •
=: ltem_5of30
? 61 � � , � I)
2 Question Id: 4320
-- - - - --- -- -
:.
- ?- Mark <J
Previous
- -- -
Next
[:>
- - - ----- - - -- - - -----
Tutorial Lab Values Notes Calculator Reverse Color Text Zoom
3 •
4 (Choice C) It is important to speak with the mother alone to get further details about the home environment, which
she may not wish to reveal in front of her daughter . However, this does not take precedence over completing the
6
7
interview with the patient and assessing for abuse.
8 (Choice D) Changes in behavior and mood often occur in normal children in response to common psychosocial
stressors such as divorce or parental discord. However, abuse must also be ruled out when a child has abrupt

11
changes in mood, bedwetting, and academic decline.
12 (Choice E) Cognitive-behavioral therapy has been studied for a number of psychological disorders in children and
13
adolescents. Al though it may have a role in the long-term management of this patient, psychotherapy would be a
14
15
secondary priority and should not delay the urgent need to exclude abuse.
16 Educational objective:
17
Physicians should have a high suspicion for abuse in children with sudden changes in mood, behavior, or
18
19 academic work, as well as in children with stressful family environments or parents with active drug/alcohol abuse.
20
21 References
22
23 • Audit of child maltreatment medical assessments in a culturally diverse, metropolitan setting.
24
• Childhood sexual abuse: identification, screening, and treatment recommendations in primary care settings.
25
26 Copyright© UWo�d. All rigllts reserved.
27 •
Block Time Remaining: 00:44:19
28
29 TIMEDTUTOR

03:39 �
T• l/vl l/l •
1M
1 •
=: ltem_6of30
? 61 � � , � I)
2 Question Id: 3235
:.
- ?Mark <J
Previous Next
[:> Tutorial Lab Values Notes Calculator Reverse Color Text Zoom
3 •
4 A 12-year-old girl is brought to the clinic by her parents for evaluation of decreased exercise tolerance and
5
increased lethargy. Her parents say she becomes short of breath with minimal exertion, and for the past 2 weeks

7
her gums have bled when she brushes her teeth. Temperature is 36. 7 C (98 F); blood pressure and pulse are
8 normal. Physical examination demonstrates pallor and hepatosplenomegaly. Laboratory results are as follows:

Complete blood count



11 Hemoglobin 6.8 g/dL
12
13 Platelets 20,000/mm3
14
Leukocytes 40,000/mm3
15
16 Blast cells 15%
17
18 An extensive workup indicates that the patient has acute lymphoblastic leukemia, and the parents are informed
19 that she will need chemotherapy. The parents do not consent to the treatment plan because they believe that the
20
side effects will be too severe and that their child's quality of life will be poor. Despite an ethics consultation and a
21
22 lengthy discussion about the benefits of treatment and likely consequences if therapy is wi thheld, the parents insist
23 on taking her home without scheduling follow-up treatment. Which of the following is the most appropriate next
24 step in the management of this patient?
25
26 0 A. Comply with the parents' wishes
27
� - -- - - - ·- -
Block Time Remaining: 00:44:16
28
29 TIMEDTUTOR

03:39 �
T• l/vl l/1 •
1M
1 �
=: ltem_6of30
? 61 � � , � I)
Question Id: 3235
:.
- ?Mark <J
Previous Next Tutorial Lab Values Notes Calculator Reverse Color Text Zoom
[:>
. . .. . . . .
2
-
3
4
Platelets 20,000/mm3
5
Leukocytes 40,000/mm3
7
8 Blast cells 15%

An extensive workup indicates that the patient has acute lymphoblastic leukemia, and the parents are informed

11 that she will need chemotherapy. The parents do not consent to the treatment plan because they believe that the
12 side effects will be too severe and that their child's quality of life will be poor. Despite an ethics consultation and a
13 lengthy discussion about the benefits of treatment and likely consequences if therapy is withheld, the parents insist
14
on taking her home without scheduling follow-up treatment. Which of the following is the most appropriate next
15
16
step in the management of this patient?
17
18 0 A. Comply with the parents' wishes
19
20
0 B. Obtain a court order for chemotherapy
21 O C. Proceed with chemotherapy immediately
O D. Provide supportive treatment only
22
23
24
O E. Recommend transfer of care to another provider
25
26
27
Submit
Block Time Remaining: 00:44:14
28
29 TIMEDTUTOR

03:39 �
T• l/vl l/1 •
1M
1 �
=: ltem_6of30
? 61 � � , � I)
Question Id: 3235
:.
- ?Mark <J
Previous Next Tutorial Lab Values Notes Calculator Reverse Color Text Zoom
[:>
. . .. . . . .
2
-
3
4
Platelets 20,000/mm3
5
Leukocytes 40,000/mm3
7
8 Blast cells 15%

An extensive workup indicates that the patient has acute lymphoblastic leukemia, and the parents are informed

11 that she will need chemotherapy. The parents do not consent to the treatment plan because they believe that the
12 side effects will be too severe and that their child's quality of life will be poor. Despite an ethics consultation and a
13 lengthy discussion about the benefits of treatment and likely consequences if therapy is withheld, the parents insist
14
on taking her home without scheduling follow-up treatment. Which of the following is the most appropriate next
15
16
step in the management of this patient?
17
18
X@ A. Comply with the parents' wishes [14%]
19 � B. Obtain a court order for chemotherapy [62%]
20
21 C. Proceed with chemotherapy immediately [13%>]
22
23 D. Provide supportive treatment only [9%]
24
E. Recommend transfer of care to another provider [0%]
25
26
27
28
29
1M
1 •
=: ltem_6of30
? 61 � � , � I)
2 Question Id: 3235
:.
- ?Mark <J
Previous Next
[:> Tutorial Lab Values Notes Calculator Reverse Color Text Zoom
3 •
Although parents generally have the r ight to make medical decisions for their minor children, parents cannot
4
5 refuse life-saving treatment for a child. In this case, the parents are refusing a proven life-saving treatment for
their child. Treatment of acute lymphoblastic leukemia currently offers a survival rate of over 80°/o. Without
7 treatment, acute lymphoblastic leukemia commonly results in death.
8
The physician should continue to engage the parents, explaining the benefits of treatment and the consequences
of withholding diagnostic tests and chemotherapy. The hospital ethics committee, social services, and hospital risk

11 management can also assist. In some cases, this multidisciplinary approach will enable parents to overcome their
12 fears and consent to the proposed treatment. However, if the parents continue to refuse life-saving treatment, the
13
physician should seek a court order to proceed with the necessary intervention.
14
15 (Choice A) Complying with parental wishes is not in the best interest of this child. Therefore, the physician has an
16
ethical duty to advocate for the child and petition the court to intervene in favor of life-saving chemotherapy.
17
18 (Choice C) If a delay for consent is life-threatening, emergency treatment for the child is legally authorized
19 regardless of the parents' wishes. However, this is a nonemergency situation, so authorization should be sought
20
through legal channels.
21
22 (Choice D) Providing only supportive care is inadequate treatment for this girl's leukemia and would likely result in
23 death. Therefore, the physician should petition the court to intervene.
24
25 (Choice E) Transfer of care to another provider would delay pursuing a court order for life-saving chemotherapy.
26
Educational objective:
27
. - ...-.. - --· - •· · ·· • - • - -· .. -. ..-. . . - . . .- ..
Block Time Remaining: 00:44:11
28
29 TIMEDTUTOR

03:40�
T• l/vl l/1 •
1M
1 •
=: ltem_6of30
? 61 � � , � I)
2 Question Id: 3235
- --- - --
:.
- ? Mark
- -- ----
<J [:>
Previous
-- - -----
Next
- - - --- - --- ---
Tutorial Lab Values
- -- - -- - - - - - -- -
Notes Calculator
-
Reverse Color Text Zoom
3 •
4 fears and consent to the proposed treatment. However, if the parents continue to refuse life-saving treatment, the
5 physician should seek a court order to proceed with the necessary intervention.

7
(Choice A) Complying with parental wishes is not in the best interest of this child. Therefore, the physician has an
8 ethical duty to advocate for the child and petition the court to intervene in favor of life-saving chemotherapy.

(Choice C) If a delay for consent is life-threatening, emergency treatment for the child is legally authorized

11
regardless of the parents' wishes. However, this is a nonemergency situation, so authorization should be sought
12 through legal channels.
13
(Choice D) Providing only supportive care is inadequate treatment for this girl's leukemia and would likely result in
14
15
death. Therefore, the physician should petition the court to intervene.
16 (Choice E) Transfer of care to another provider would delay pursuing a court order for life-saving chemotherapy.
17
18 Educational objective:
19 In a nonemergency situation in which a parent refuses potentially life-saving treatment for their child, the physician
20
should seek a court order mandating treatment.
21
22
23 References
24
• Parents know best: or do they? Treatment refusals in paediatric oncology.
25
26 Copyright© UWo�d. All rigllts reserved.
27 •
Block Time Remaining: 00:44:11
28
29 TIMEDTUTOR

03:40�
T• l/vl l/l •
1M
1 •
=: ltem_7of30
? 61 � � , � I)
2 Question Id: 3237
:.
- ?Mark <J
Previous Next
[:> Tutorial Lab Values Notes Calculator Reverse Color Text Zoom
3 •
4 A 34-year-old man calls the office requesting an urgent appointment due to cough and cold symptoms. His
5
temperature is 37.2 C (99 F), but he feels "miserable" and is unsure if he should go to work the next day. The
6
patient has a history of seasonal allergies and knee pain from a past sports injury. He has no other medical
8 problems but does have a history of frequent office visits. The receptionist informs him that the physician is
preparing to leave as it is near closing time but that an appointment is available for the next morning. The patient
� declines the offer of an appointment and hangs up. As the physician leaves the office 30 minutes later to make
11
evening rounds at the hospital, the patient arrives and insists on being seen immediately. He sounds congested
12
and coughs occasionally but is breathing normally. He has no other symptoms. He again asks to be examined.
13
14 Which of the following is the most appropriate response?
15
16 O A. "Although I understand your concern, we should address it tomorrow because it is not an emergency."
17
18
O B. "I am sorry. I would normally see you, but I have another appointment that can't wait."
19 O C. "I am sorry you drove all this way, but your problem can wait until tomorrow."
20
21 O D. "I insist that you go to the nearest emergency department for evaluation."
22
23
O E. "I regret that I cannot see you now; didn't my staff inform you that it is closing time?"
24 O F. "Let me take a quick look; it's probably nothing to worry about."
25
26
27 Submit
Block Time Remaining: 00:44:09
28
29 TIMEDTUTOR

03:40�
T• l/vl l/1 •
1M
1 •
=: ltem_7of30
? 61 � � , � I)
2 Question Id: 3237
:.
- ?Mark <J
Previous Next
[:> Tutorial Lab Values Notes Calculator Reverse Color Text Zoom
3 •
4 A 34-year-old man calls the office requesting an urgent appointment due to cough and cold symptoms. His
5
temperature is 37.2 C (99 F), but he feels "miserable" and is unsure if he should go to work the next day. The
6
patient has a history of seasonal allergies and knee pain from a past sports injury. He has no other medical
8 problems but does have a history of frequent office visits. The receptionist informs him that the physician is
preparing to leave as it is near closing time but that an appointment is available for the next morning. The patient
� declines the offer of an appointment and hangs up. As the physician leaves the office 30 minutes later to make
11
evening rounds at the hospital, the patient arrives and insists on being seen immediately. He sounds congested
12
and coughs occasionally but is breathing normally. He has no other symptoms. He again asks to be examined.
13
14 Which of the following is the most appropriate response?
15
16 1' A. "Although I understand your concern, we should address it tomorrow because it is not an emergency."
17 [62%]
18
B. "I am sorry. I would normally see you, but I have another appointment that can't wait." [7%]
19
20 X(' C. "I am sorry you drove all this way, but your problem can wa it until tomorrow." [2%]
21
22 D. "I insist that you go to the nearest emergency department for evaluation." [11%]
23
E. "I regret that I cannot see you now; didn't my staff inform you that it is closing time?" [7%>]
24
25 F. "Let me take a quick look; it's probably nothing to worry about." [8%]
26
27
28
29
1M
1 •
=: ltem_7of30
? 61 � � , � I)
2 Question Id: 3237
:.
- ?Mark <J
Previous Next
[:> Tutorial Lab Values Notes Calculator Reverse Color Text Zoom
3 •
This physician is faced with a patient who inappropriately demands to be seen after hours for a non-urgent
4
5 concern. A stable patient who fails to arrive at the appointed time should be advised to reschedule the
6 appointment; the physician is under no obligation to immediately see this patient. Performing an examination after
hours is inappropriate and reinforces the patient's sense of entitlement and poor boundaries. Acquiescing to the
8
patient's demands would only encourage similar behavior in the future and create further tension in the physician­
patient relationship due to the patient's lack of respect for the physician's time (Choice F).

11 The most appropriate response is to calmly set limits and establish clear professional boundaries. The
12
physician should explain in a polite yet firm tone that the examination will be deferred until the next day because
13
the condition is not an emergency.
14
15 (Choice B) This response suggests that the patient's behavior is appropriate and offers an excuse not to see the
16
patient rather than explaining that non-emergency concerns can wait.
17
18 (Choices C and E) These responses set limits but are dismissive of the patient's concerns. It is better to
19 acknowl edge the concerns but explain that non-urgent issues are addressed during regular office hours.
20
21 (Choice D) Recommending that the patient go to the emergency department for a non-urgent condition is
22 inappropriate and places an unnecessary burden on emergency services.
23
24
Educational objective:
25 Physicians should respond politely but firmly to inappropriate patient requests. Maintaining professional
26 boundaries is an important component of the physician-patient relationship.
27 •
Block Time Remaining: 00:44:06
28
29 TIMEDTUTOR

03:40�
T• l/vl l/1 •
1M
1 •
=: ltem_8of30
? 61 � � , � I)
2 Question Id: 3805
:.
- ?Mark <J
Previous Next
[:> Tutorial Lab Values Notes Calculator Reverse Color Text Zoom
3 •
4 A 35-year-old woman established primary care at a clinic 3 months ago. Last month, she arrived unannounced,
5
urgently requesting to see the same physician. Due to a cancellation, she was seen later that day for the
6
7
complaint of a rash on her chest. The male physician completed a thorough but unremarkable physical
examination in the presence of a female nurse pract itioner. Two weeks later, the patient comes to the same
9 physician's office at closing time and wi thout an appointment. She complains of "needing to talk to the doctor
10 immediately about a private matter." She informs the receptionist that it is "absolutely critical" for her to be seen
11
and examined for similar skin complaints that seem to "come and go" and "itch frequently. " The patient is calm but
12
insists that an appointment with the female nurse practitioner is not acceptable and instead requests to see the
13
14 physician privately, without the presence of another staff member. Which of the following would be the most
15 appropriate initial response by the physician?
16
17 O A. Ask the patient to have a seat in the office and proceed with the unscheduled appointment.
O B. Have the receptionist contact security to remove the patient from the premises.
18
19
20
21
O C. Have the receptionist instruct the patient to schedule an appointment during normal office hours.
22 O D. Instruct the receptionist to inform the patient that she must leave and not return to the clinic.
O E. Transfer the patient's care to a partner who is on call for the evening and available by phone.
23
24
25
26
27 Submit

Block Time Remaining: 00:44:04


28
29 TIMEDTUTOR

03:40�
T• l/vl l/1 •
1M
1 •
=: ltem_8of30
? 61 � � , � I)
2 Question Id: 3805
:.
- ?Mark <J
Previous Next
[:> Tutorial Lab Values Notes Calculator Reverse Color Text Zoom
3 •
4 A 35-year-old woman established primary care at a clinic 3 months ago. Last month, she arrived unannounced,
5
urgently requesting to see the same physician. Due to a cancellation, she was seen later that day for the
6
7
complaint of a rash on her chest. The male physician completed a thorough but unremarkable physical
examination in the presence of a female nurse pract itioner. Two weeks later, the patient comes to the same
9 physician's office at closing time and wi thout an appointment. She complains of "needing to talk to the doctor
10 immediately about a private matter." She informs the receptionist that it is "absolutely critical" for her to be seen
11
and examined for similar skin complaints that seem to "come and go" and "itch frequently. " The patient is calm but
12
insists that an appointment with the female nurse practitioner is not acceptable and instead requests to see the
13
14 physician privately, without the presence of another staff member. Which of the following would be the most
15 appropriate initial response by the physician?
16
17 A. Ask the patient to have a seat in the office and proceed with the unscheduled appointment. [5%>]

X r B. Have the receptionist contact security to remove the patient from the premises. [0%]
18
19
20
� C. Have the receptionist instruct the patient to schedule an appointment during normal office hours. [87%>]
21
22 D. Instruct the receptionist to inform the patient that she must leave and not return to the clinic. [0%]
23
24 E. Transfer the patient' s care to a partner who is on call for the evening and available by phone. [6%]
25
26
27
- I I I:
Block Time Remaining: 00:44:01
28
29 TIMEDTUTOR

03:40�
T• l/vl l/1 •
1M
1 •
=: ltem_8of30
? 61 � � , � I)
2 Question Id: 3805
:.
- ?Mark <J
Previous Next
[:> Tutorial Lab Values Notes Calculator Reverse Color Text Zoom
3 •
4 This question addresses the importance of maintaining professional conduct when dealing with patients of all
5 types, ranging from hostile to seductive. Several actions suggest that this patient could have boundary issues.
6
These include:
7
• Arrival at unscheduled times and/or at closing time (when others are less likely to be available)
9 • Insistence on seeing the same physician for each visit (for mild conditions) and in private
10
• Frequent return visits for nonspecific complaints
11
• Health complaints that necessitate examination of private areas or undressing, despite recent normal findings
12
13 To maintain a therapeutic distance, the physician should have the receptionist politely inform the woman that
14
patients are seen only when they have scheduled appointments during normal office hours and with the presence
15
of another staff member.
16
17 (Choice A) Having an unscheduled appointment with the patient under these circumstances would only blur the
18
doctor-patient relationship boundary. Instead, the physician should maintain a professional demeanor and discuss
19
20
the patient's medical concerns during a scheduled appointment.
21 (Choice B) Although the patient has inappropriate expectations, the situation does not appear to involve imminent
22
concern for impending violence or escalation of her demands. Thus, contacting security is not likely to be
23
24
necessary.
25 (Choice D) The correct approach, even for seductive patients, is to provide the required medical care in a
26
professional manner . This is not an emergency, so having the patient return for a scheduled visit, as opposed to
27

Block Time Remaining: 00:44:01


28
29 TIMEDTUTOR

03:40�
T• l/vl l/1 •
1M
1 •
=: ltem_8of30
? 61 � � , � I)
2 Question Id: 3805
:.
- ?Mark <J
Previous Next
[:> Tutorial Lab Values Notes Calculator Reverse Color Text Zoom
3 •
4 To maintain a therapeutic distance, the physician should have the receptionist politely inform the woman that
5 patients are seen only when they have scheduled appointments during normal office hours and wi th the presence
6 of another staff member.
7
(Choice A) Having an unscheduled appointment with the patient under these circumstances would only blur the
9 doctor-patient relationship boundary. Instead, the physician should maintain a professional demeanor and discuss
10 the patient's medical concerns during a scheduled appointment.
11
12 (Choice B) Although the patient has inappropriate expectations, the situation does not appear to involve imminent
13 concern for impending violence or escalation of her demands. Thus, contacting security is not likely to be
14 necessary.
15
16 (Choice 0) The correct approach, even for seductive patients, is to provide the required medical care in a
17 professional manner . This is not an emergency, so having the patient return for a scheduled visit, as opposed to
18 telling her she is not to come back, would be appropriate.
19
20 (Choice E) It is the end of the work day, and there is likely no need for the partner to take any immediate action.
21 Thus, transferring care is not indicated.
22
23 Educational objective:
24 When dealing with difficult patients, the physician must maintain professional conduct and responsibilities while
25 addressing their medical and psychological needs.
26
Copyright© UWo�d. All rigllts rese,ved.
27 •
Block Time Remaining: 00:44:01
28
29 TIMEDTUTOR

03:40�
T• l/vl l/1 •
1M
1 •
=: ltem_9of30
? 61 � � , � I)
2 Question Id: 10660
:.
- ?Mark <J
Previous Next
[:> Tutorial Lab Values Notes Calculator Reverse Color Text Zoom
3 •
4
5 The following vignette applies to the next 2 items. The items in the set must be answered in sequential order.
6 Once you click Proceed to Next Item, you wi ll not be able to add or change an answer.

..
7
8 Item 1 of 2

A tertiary care hospital undergoes an independent review of malpractice claims related to missed or delayed

11
diagnoses that have occurred over the past 10 years in the emergency department. A large proportion of these
12 errors led to serious harm, with approximately half resulting in death. The review concludes that the rate of these
13 mistakes in the hospital's emergency department is 25% higher than in comparable hospitals nationwide. The
14 hospital administration convenes an internal committee to address this issue. Which of the following is the most
15
likely underl ying cause of the majority of these medical errors?
16
17
18
O A. Communication failures between providers
19 O B. High emergency department acuity
20
21 0 C. High emergency department volume
22
23
O D. Lack of computerized patient handoff tools
24 O E. Lack of detailed signout notes
25
26
27 Submit
Block Time Remaining: 00:43:59
28
29 TIMEDTUTOR

03:40�
T• l/vl l/1 •
1M
1 •
=: ltem_9of30
? 61 � � , � I)
2 Question Id: 10660
:.
- ?Mark <J
Previous Next
[:> Tutorial Lab Values Notes Calculator Reverse Color Text Zoom
3 •
4
5 The following vignette applies to the next 2 items. The items in the set must be answered in sequential order.
6 Once you click Proceed to Next Item, you wi ll not be able to add or change an answer.

...
7
8 Item 1 of 2

A tertiary care hospital undergoes an independent review of malpractice claims related to missed or delayed

11
diagnoses that have occurred over the past 10 years in the emergency department. A large proportion of these
12 errors led to serious harm, with approximately half resulting in death. The review concludes that the rate of these
13 mistakes in the hospital's emergency department is 25% higher than in comparable hospitals nationwide. The
14 hospital administration convenes an internal committee to address this issue. Which of the following is the most
15
likely underl ying cause of the majority of these medical errors?
16
17
� A. Communication failures between providers [76%]
18
19 X � B. High emergency department acuity [3%]
20
21 C. High emergency department volume [9%]
22
D. Lack of computerized patient handoff tools [2%]
23
24 E. Lack of detailed signout notes [7%]
25
26
27
28
29
1M
1 •
=: ltem_9of30
? 61 � � , � I)
2 Question Id: 10660
:.
- ?Mark <J
Previous Next
[:> Tutorial Lab Values Notes Calculator Reverse Color Text Zoom
3 •
4
Discontinuity is an unfortunate but unavoidable reality of hospital care, with multiple providers assuming
5
responsibility for a patient's care at different times of the day. The process of transferring responsibility for medical
6
care is referred to as a patient handoff, with "signout" referring to the process of transmitting information about

...
7
8 the patient and needed follow-up care. Oversights and communication failures during the signout and handoff
process have been linked to adverse clinical events in a range of hospital settings. Omission of key information
during handoffs often results in near-misses, avoidable escalations in care (eg, transfer to the intensive care unit),
11
inefficiencies in care, and delays in diagnosis or treatment.
12
13 (Choices B and C) Although high patient volume or acuity can increase the probability of medical errors, these
14
are not the underl ying reasons why medical errors occur.
15
16 (Choice 0) Although computerized handoffs have become part of electronic medical record systems, there is
17 minimal evidence supporting their ability to improve handoff quality or patient outcomes.
18
19 (Choice E) Increasing the level of detail in signout notes has not been found to improve error rates stemming from
20 the patient signout process. Overly detailed notes can often lead to difficulty in locating important information in
21 emergencies or inability to see the "big picture."
22
23 Educational objective:
24 Communication failures between physicians during patient handoffs are a large contributor to medical errors and
25 adverse patient outcomes.
26
27
-. . ....
Block Time Remaining: 00:43:56
28
29 TIMEDTUTOR

03:40�
T• l/vl l/1 •
1M
1 �
=: ltem _1 0 of30
:. ? 61 � � , � 0
2 Question Id: 10661 - ?Mark <J
Previous Next
[:> Tutorial Lab Values Notes Calculator Reverse Color Text Zoom
3
4 Item 2 of 2
5
6 Following a prolonged investigation, the hospital determines that the majority of the missed or delayed emergency
department diagnoses are directl y linked to communication failures among physicians during the signout process.

-
7
8 Which of the following has been found to most effectively improve communication of relevant information during
patient transfers?

11
12
O A. Implementing a signout checklist

13 O B. Inclusion of nurses and social workers in the signout process

O C.
14
15 Increasing the length of time allotted for signout
16
17
O D. Mandating a face-to-face signout procedure

18 0 E. Minimizing interruptions
19
20
21 Submit
22
23
24
25
26
27

Block Time Remaining: 00:43:55


28
29 TIMEDTUTOR

03:40�
T• l/vl l/1 •
1M
1 •
=: ltem _1 0 of30
:. ? 61 � � , � 0
2 Question Id: 10661 - ?Mark <JPrevious Next
[:> Tutorial Lab Values Notes Calculator Reverse Color Text Zoom
3 •
4 Item 2 of 2
5
6 Following a prolonged investigation, the hospital determines that the majority of the missed or delayed emergency
department diagnoses are directl y linked to communication failures among physicians during the signout process.

--
7
8 Which of the following has been found to most effectively improve communication of relevant information during
patient transfers?

11
� A. Implementing a signout checklist [55%]
12
13 B. Inclusion of nurses and social workers in the signout process [9%]
14
15 C. Increasing the length of time allotted for signout [1 %]
16
X r D. Mandating a face-to-face signout procedure [27%]
17

1
18 E. Minimizing interruptions [6%]
19
20
21
1 nco!J!ect
22 l1o. 55% II\ 18 Seconds i==l 08/20/2018
23
Corr�t answer L!!!. Answere<I correctly '-:::.J
TimeSpent I.!!!.! Last Update<!
A
24
25
26 Explanation
27 •
Block Time Remaining: 00:43:38
28
29 TIMEDTUTOR
1M
1 •
=: ltem _1 0 of30
:. ? 61 � � , � 0
2 Question Id: 10661 - ?Mark <J
Previous Next
[:> Tutorial Lab Values Notes Calculator Reverse Color Text Zoom
3 •
4 Medical errors resulting from communication failures between medical providers are most effectively addressed by
5 implementing a systematic signout process that includes checklists to improve efficacy and accuracy. Patient
6 transfers should be conducted verbally and supplemented with an updated and clear written component. Ideally,

--
7
transfers should occur face to face (with minimal interruptions), but this has not been found to be as effective as
8
implementing systematic checklists as part of the signout procedure (Choices D and E). Essential components of
an effective signout checklist include do not resuscitate/do not intubate status, hosp ital course and recent events,
11 current condition, and anticipatory information (eg, "if the patient's mental status changes, check a blood gas and
12 consider transfer back to ICU"). During the signout process, physicians should go through each of these
13
categories systematically to ensure that no important information is missed.
14
15 (Choice B) Although conducting signout with other team members could provide additional information or
16 concerns about the patient, this would be impractical. Interdisciplinary rounds would be a more appropriate setting
17
for the whole team to discuss the patient at greater length, including discharge planning and follow-up care.
18
19 (Choice C) Although physicians should designate an appropriate amount of time for patient signouts, increased
20 time for patient transfers does not necessarily lead to improved signout quality or better outcomes.
21
22 Educational objective:
23 Checklists are an important tool to prevent undesired medical outcomes that result from physician communication
24 failures during the patient handoff process.
25
26
27 References
Block Time Remaining: 00:43:38
28
29 TIMEDTUTOR
1M
1 �
=: ltemH of30
:. ? 61 � � , � 0
2 Question Id: 3792 - ?Mark <J
Previous Next
[:> Tutorial Lab Values Notes Calculator Reverse Color Text Zoom
3
4 A 32-year-old woman comes to the physician with lower back pain for the past week. She developed the pain after
5
lifting some heavy furniture. She has no other symptoms. Physical examination shows mild paraspinal muscle
6
7
tenderness. There is no radiation of pain with straight leg raise on either side, and no neurologic deficits are noted
8 in the lower extremities. However, multiple bruises of various ages are noted on her abdomen, back, and chest.
9 When the bruises are pointed out, the woman begins to cry. She is also quiet and avoids eye cont¥t. Which of
10 the following is the most appropriate initial response?

12 O A. Do you feel down, depressed, or hopeless?


13
14 O B. Do you have a safety plan to leave the situation?
15
16
O C. I am going to call the police to come investigate.
17 O D. Tell me more about the bruises on your body.
18
19 O E. These bruises are clearly from abuse and you do not deserve to be treated like this.
20
21
22 Submit
23
24
25
26
27

Block Time Remaining: 00:43:36


28
29 TIMEDTUTOR
1M
1 •
=: ltemH of30
:. ? 61 � � , � 0
2 Question Id: 3792 - ?Mark <JPrevious Next
[:> Tutorial Lab Values Notes Calculator Reverse Color Text Zoom
3 •
4 A 32-year-old woman comes to the physician with lower back pain for the past week. She developed the pain after
5
lifting some heavy furniture. She has no other symptoms. Physical examination shows mild paraspinal muscle
6
7
tenderness. There is no radiation of pain with straight leg raise on either side, and no neurologic deficits are noted
8 in the lower extremities. However, multiple bruises of various ages are noted on her abdomen, back, and chest.
9 When the bruises are pointed out, the woman begins to cry. She is also quiet and avoids eye contact. Which of
10 the following is the most appropriate initial response?

12 A. Do you feel down, depressed, or hopeless? [2%]


13
14 X@ B. Do you have a safety plan to leave the situation? [6%>]
15
C. I am going to call the police to come investigate. [0%]
16
17 D. Tell me more about the bruises on your body. [90%]
18
19 E. These bruises are clearly from abuse and you do not deserve to be treated like this. [0%]

I
20
21
22
Incorrect II\ 5 Seconds 09/21/2018
23 (1, , 90% I==!
Corre<:t answer L!!!. Answered correctly "-::.J TimeSpent l.!!!.I Last Updated
24 D
25
26
27
Explanation

Block Time Remaining: 00:43:33


28
29 TIMEDTUTOR
1M
1 •
=: ltemH of30
? 61 � � , � 0
2 Question Id: 3792
:.
- ?Mark <J
Previous Next
[:> Tutorial Lab Values Notes Calculator Reverse Color Text Zoom
3 •
4
Physicians should be alert to clues suggesting physical abuse as patients may not al ways report such incidents
5 spontaneously. Multiple bruises, recurrent fractures, or repeated visits to the physician for unexplained injuri es are
6 all signs of possible abuse. In cases of possible abuse, the physician must be able to conduct an effective patient
7 interview to provide proper care, ensure patient safety, and fulfill legal reporting requirements.
8
9 The initial statement in such cases should be made tactfully and take into account the patient's personality,
10 emotional state, socioeconomic and cultural background, and the broader physician-patient relationship. Open­
ended questions and general statements of observation usually provide the best introduction to such a discussion
12
and allow patients to discuss the issues in their own words. Acknowledging the patient's feelings and/or
13
14
encouraging further dialogue about the bruises would be good starting points.
15 (Choice A) Patients who are victims of abuse may have significant associated emotional symptoms, whether as a
16
result of the abuse itself, from pre-existing affective illness, or from concurrent substance abuse. However, the
17
18 physician should first gain an understanding of the abuse situation and then ensure immediate patient safety.
19 Once immediate safety has been addressed, the physician and patient can discuss what steps may be taken to
20 address concurrent affective symptoms.
21
22 (Choice B) At some point in the conversation, the physician should discuss a safety plan with the patient.
23 However, before making specific recommendations, it is necessary to understand the nature of the possible abuse
24 and explore what options she may have already considered.
25
26 (Choice C) Advising the patient to consider reporting the incident to the police may be appropriate in cases of
27 suspected abuse, but this is not a good opening statement. The prospect of filing charges may frighten the patient •
Block Time Remaining: 00:43:33
28
29 TIMEDTUTOR
1M
1 •
=: ltemH of30
? 61 � � , � 0
2 Question Id: 3792
:.
- ?Mark <J
Previous Next
[:> Tutorial Lab Values Notes Calculator Reverse Color Text Zoom
3 •
address concurrent affective symptoms.
4
5 (Choice B) At some point in the conversation, the physician should discuss a safety plan with the patient.
6 However, before making specific recommendations, it is necessary to understand the nature of the possible abuse
7
and explore what options she may have already considered.
8
9 (Choice C) Advising the patient to consider reporting the incident to the police may be appropriate in cases of
10 suspected abuse, but this is not a good opening statement. The prospect of filing charges may frighten the patient
into denying the abuse and discourage a more thorough discussion of the issue.
12
13 (Choice E) Although the physician may suspect abuse, it is important not to make unfounded assumptions. The
14 possibility of abuse should be explored in an open and caring conversation with the patient. Value judgments, if
15
any, should originate from the patient rather than the physician.
16
17 Educational objective:
18
Patients who are victims of suspected physical abuse should be approached with empathic interviewing
19
techniques. The physician should ask open-ended questions to allow patients to describe their situation on their
20
21 own terms. The first priorities are to obtain an accurate and thorough understanding of the abuse and take any
22 necessary action to ensure patient safety. Additional steps may then be needed to address concurrent emotional
23 symptoms and satisfy legal reporting requirements.
24
25
26 References
27
artner violence and abuse of elderl and vulnerable adults: U.S. reventive services
Block Time Remaining: 00:43:33
28
29 TIMEDTUTOR
1M
1 •
=: ltem _1 2 of30
:. ? 61 � � , � 0
2 Question Id: 2642 - ?Mark <J
Previous Next
[:> Tutorial Lab Values Notes Calculator Reverse Color Text Zoom
3 •
4 A 48-year-old man with an extensive smoking history is evaluated for cough and an episode of mild hemoptysis.
5
After the patient ignored his symptoms for weeks, his wife finally convinced him to make an appointment. Chest x­
6
7
ray reveals a mass lesion in the right upper lobe. A chest CT scan and bronchoscopy with biopsy are scheduled
8 for that afternoon, and the patient is instructed to schedule a follow-up visit with the physician 3 days later to
discuss the results. The patient agrees to have the tests done but declines the follow-up appointment, saying that
� he and his wife are leaving for a previously scheduled week-long anniversary celebration and vacation the next
11
day. Despite education about diagnostic possibilities, he is convinced that his symptoms are stress related and
that he will feel better by the time he returns. The physician is highly concerned that the workup may show lung
13
14 cancer although the patient clinically appears to be stable. Which of the following is the most appropriate response
15 by the physician?
16
17 O A. I am concerned that you may be in denial about the seriousness of your condition.

O B.
18
19
I recommend that you postpone your trip because there is a high probability that this is cancer.
20
21
O C. I recommend that you postpone your trip so that we can discuss your test results.

22 O D. Please give me permission to speak to your wi fe.

O E.
23
24
Please leave your information and I will contact you by phone with the results.
25
26
O F. Please set up an appointment as soon as you return so we can discuss your results.

27 •
Block Time Remaining: 00:43:31
28
29 TIMEDTUTOR
1M
1 �
=: ltem _1 2 of30
:. ? 61 � � , � 0
2 Question Id: 2642
-
- ?Mark <JPrevious Next
[:>
f:f
Tutorial Lab Values Notes
-
Calculator

Reverse Color Text Zoom
.. - •- - • • -• • • • J:t: - - , - - ' • ' -• • .. ' - - -••• -
3 '

4 ray reveals a mass lesion in the right upper lobe. A chest CT scan and bronchoscopy with biopsy are scheduled
5 for that afternoon, and the patient is instructed to schedule a follow-up visit with the physician 3 days later to
6
discuss the results. The patient agrees to have the tests done but declines the follow-up appointment, saying that
7
8 he and his wife are leaving for a previously scheduled week-long anniversary celebration and vacation the next
day. Despite education about diagnostic possibilities, he is convinced that his symptoms are stress related and

that he will feel better by the time he returns. The physician is highly concerned that the workup may show lung
11 cancer although the patient clinically appears to be stable. Which of the following is the most appropriate response
by the physician?
13
14
X (i' A. I am concerned that you may be in denial about the seriousness of your condition. [14%]
15
16 B. I recommend that you postpone your trip because there is a high probability that this is cancer. [3%]
17
18 C. I recommend that you postpone your trip so that we can discuss your test results. [25%]
19
D. Please give me permission to speak to your wi fe. [0%>]
20
21 E. Please leave your information and I will contact you by phone with the results. [5%]
22
23 F. Please set up an appointment as soon as you return so we can discuss your results. [50%]
24
25
26
lncor�ct
27 (1, , 50%
L!!!. Answered correctl II • •I
Block Time Remaining: 00:43:26
28
29 TIMEDTUTOR
1M
1 •
=: ltem _1 2 of30
:. ? 61 � � , � 0
2 Question Id: 2642 - ?Mark <J
Previous Next
[:> Tutorial Lab Values Notes Calculator Reverse Color Text Zoom
3 •
In this situation, it is highly possible that the patient will subsequently receive bad news about his health from the
4
5 physician. The patient is in denial and his condition may be serious (ie, probable lung cancer). However, he
6 appears to be clinically stable and a week's delay is unlikely to affect his treatment outcome. The best approach is
7 to respect the patient's defenses at this time and arrange for a follow-up on his return.
8
When delivering bad news, a face-to-face visit in a comfortable private setting is preferred as it allows the
physician to respond to nonverbal aspects of communication and provide empathy and emotional support as

11 needed. It is also important to assess the patient's understanding of the condition and how much the patient
actually wants to know. Other steps include gaining an understanding of cultural/educational/reli gious issues,
13
making medical information understandable to the patient, and formulating a collaborative treatment plan (SPIKES
14
15 protocol).
16
(Choice A) Attributing his symptoms to stress may be the patient's initial way of coping with expected bad news.
17
It would be inappropriate to confront the patient with his denial at this point. If and when a cancer diagnosis is
18
19 confirmed, the news should be delivered in a sensitive manner.
20
(Choice B) It is premature and insensitive to provide the patient with a "probable" diagnosis of cancer before the
21
22
workup is complete. Not having a definitive diagnosis is likely to cause uncertainty and unnecessary alarm.
23 (Choice C) The benefits of the patient going on what might be his last vacation outweigh the minimal risks of
24
delaying his treatment for a week. In additi on, asking the patient to postpone his vacation in the face of his denial,
25
26
without clear evidence of a diagnosis, may negatively affect his future engagement with the physician.
27 It is premature and inappropriate to bypass communication with the patient and call his spouse. All •
Block Time Remain,...,. VV•-•4V
28
29 TIMEDTUTOR
1M
1 �
=: ltem _1 2 of30
:. ? 61 � � , � 0
2 Question Id: 2642 - ?Mark <J Previous Next
[:> Tutorial Lab Values Notes Calculator Reverse Color Text Zoom
3 t I t I

4
(Choice A) Attributing his symptoms to stress may be the patient's initial way of coping with expected bad news.
5
6 It would be inappropriate to confront the patient with his denial at this point. If and when a cancer diagnosis is
7 confirmed, the news should be delivered in a sensitive manner.
8
(Choice B) It is premature and insensitive to provide the patient with a "probable" diagnosis of cancer before the
workup is complete. Not having a definitive diagnosis is likely to cause uncertainty and unnecessary alarm.

11 (Choice C) The benefits of the patient going on what might be his last vacation outweigh the minimal risks of
delaying his treatment for a week. In addition, asking the patient to postpone his vacation in the face of his denial,
13
14 without clear evidence of a diagnosis, may negatively affect his future engagement with the physician.
15
(Choice 0) It is premature and inappropriate to bypass communication with the patient and call his spouse. All
16
preliminary communication should still be directed to the patient.
17
18 (Choice E) Contacting the patient by phone would not be the most appropriate method for communicating serious
19
news. Face-to-face meetings are more appropriate for delivering bad news.
20
21 Educational objective:
22 Delivering bad news is optimally done in a face-to face meeting that allows the physician to better assess the
23
patient's emotional reactions and provide empathy and necessary support.
24
25
26 References
27
... II -.. ei. e e ... I e e ... e I ...a. • • ,.. ... I • I ,.. I ...

Block Time Remaining: 00:43:26


28
29 TIMEDTUTOR
1M
1 •
=: ltem _1 3 of30
:. ? 61 � � , � 0
2 Question Id: 2197 - ?Mark <J
Previous Next
[:> Tutorial Lab Values Notes Calculator Reverse Color Text Zoom
3 •
4 A 57-year-old woman comes to the emergency department experiencing severe lower back pain. The pain began
5 4 weeks ago and is getting progressively worse. At night, the pain has become so severe that it awakens her from
6 sleep. The patient has tried over-the-counter ibuprofen with no relief of symptoms. She also reports feeling more
7
tired than usual but says, "It is probably because my kids are home for the holidays and are keeping me busy."
8
The patient has a history of breast cancer that has been in remission for 2 years. Her physical examination is
significant for weakness and numbness of the left lower limb. Radiographic findings are consistent with metastatic

11 lesions in her spine. Which of the following is the most appropriate next statement in initiating a conversation
12 about this patient's condition?

14 O A. "Have you considered the possibility that your pain is due to metastatic disease?"
15
16 O B. "Have you considered whether or not you want to know the cause of your condition?"
17
18
O C. "I am sorry but the examination suggests the cancer may have returned. I can imagine this is
19
devastating news."
20 O D. "I am sorry to tell you that the cancer has possibly spread, but we can treat you aggressively with
21
radiation and chemotherapy. "
22
23 O E. "What is your understanding about what might be causing your back pain?"
24
25
26 Submit
27 •
Block Time Remaining: 00:43:25
28
29 TIMEDTUTOR
1M
1 •
=: ltem_1 3of30
? 61 � � , � 0
2 Question Id: 2197
:.
- ?Mark <J
Previous Next
[:> Tutorial Lab Values Notes Calculator Reverse Color Text Zoom
3 •
4 A 57-year-old woman comes to the emergency department experiencing severe lower back pain. The pain began
5
4 weeks ago and is getting progressively worse. At night, the pain has become so severe that it awakens her from
6
7
sleep. The patient has tried over-the-counter ibuprofen with no relief of symptoms. She also reports feeling more
8 tired than usual but says, "It is probably because my kids are home for the holidays and are keeping me busy."
The pati ent has a history of breast cancer that has been in remission for 2 years. Her physical examination is
� significant for weakness and numbness of the left lower limb. Radiographic findings are consistent with metastatic
11
lesions in her spine. Which of the following is the most appropriate next statement in initiating a conversation
12
about this patient's condition?
14
A. "Have you considered the possibility that your pain is due to metastatic disease?" [0%]
15
16 B. "Have you considered whether or not you want to know the cause of your condition?" [2%]
17
18 C. "I am sorry but the examination suggests the cancer may have returned. I can imagine this is
19 devastating news." [12%]
20
21
X � D. "I am sorry to tell you that the cancer has possibly spread, but we can treat you aggressively with
22 radiation and chemotherapy. " [2%]
23 E. "What is your understanding about what might be causing your back pain?" [81 %>]
24
25
26
27 Incorrect
- .....
a7a'T7'T-T.1

Block Time Remaining: 00:43:22


28
29 TIMEDTUTOR

03: 42 �
T• l/vl l/1 •
1M
1 •
=: ltem_1 3of30
? 61 � � , � 0
2 Question Id: 2197
:.
- ?Mark <J
Previous Next
[:> Tutorial Lab Values Notes Calculator Reverse Color Text Zoom
3 •
4 Breaking bad news is one of the most difficult situations a physician will encounter. A sensitive, patient-centered
5 approach that is responsive to the patient's needs is preferred rather than expressing excessive sympathy or
6 prematurely suggesting treatment options. An important first step is to find out how much the patient already
7
knows and what the patient believes about his or her condition. This patient has a previous diagnosis of breast
8
cancer, but to what extent she has been informed or understands her risk for metastatic disease is unclear. First
listening to how the patient discusses her condition and fears will help the physician p hrase the information

11 accordingly. The physician can then communicate his or her suspicions and concerns in an empathic manner
12 using language that is understandable to the patient, considering any religious or cultural issues, and adjusting the
level of detail according to how much the patient wants to know.
14
15 Let the patient ask questions but do not overload him or her with too much information. Information may need to
16 be repeated later after the emotion of the situation has been absorbed. Although each situation is different and the
17
physician's response should be guided by the patient, a protocol such as SPIKES can be used for general
18
guidance.
19
20 (Choice A) Immediately stating the likely diagnosis without any preparation or assessment of this patient's
21
perception is not an optimal way to begin the discussion. Using simpler vocabulary rather than medical terms such
22
as "metastatic disease" may also be preferable.
23
24 (Choice B) Physicians should generally determine how much the patient wants to know. However, beginning the
25
discussion this way suggests bad news and would likely raise this patient's anxiety level, especially if she has not
26
27
considered metastases as a possibility.

Block Time Remaining: 00:43:22
28
29 TIMEDTUTOR

03: 42 �
T• l/vl l/1 •
1M
1 �
=: ltem_1 3of30
? 61 � � , � 0
2 Question Id: 2197
:.
- ?Mark <J
Previous Next
[:> Tutorial Lab Values Notes Calculator Reverse Color Text Zoom
3 p p p y • g • I g p • ry •
4 as "metastatic disease" may also be preferable.
5
6 (Choice B) Physicians should generally determine how much the patient wants to know. However, beginning the
7 discussion this way suggests bad news and would likely raise this patient's anxiety level, especially if she has not
8 considered metastases as a possibility.

(Choice C) This approach delivers the news abruptly and then expresses excessive sympathy without allowing

11 this patient to express her own emotions. This approach can engender hopelessness and hinder appropriate
12 information exchange.

14 (Choice 0) Stating that this patient likely has cancer in her back and then jumping straight to treatment options is
15 insensitive and premature. The physician should pause after delivering bad news and allow the patient to express
16 emotion and ask questions.
17
18 Educational objective:
19 When delivering potentially bad news, a physician should first assess the patient's comprehension, fears, and
20 desire for information to deliver the news in an empathetic yet comprehensible manner.
21
22
23 References
24
• Delivering bad news to patients.
25
26 • Breaking bad news education for emergency medicine residents: a novel training module using simulation
27 with the SPIKES protocol. •
Block Ti..... ,,o;;..,........." . .......-......
28
29 TIMEDTUTOR
1M
1 �
=: ltem _1 4of30
:. ? 61 � � , � 0
2 Question Id: 8957 - ?Mark <J
Previous Next
[:> Tutorial Lab Values Notes Calculator Reverse Color Text Zoom
3
4 A 4th-year medical student working in the intensive care unit is helping to care for a 75-year-old man with end­
5
stage lung cancer. The patient suffers a cardiac arrest and, despite intensive resuscitation and emergency
6
7
management, cannot recover and is declared dead 15 minutes later. Team members provide emotional support to
8 his family. The supervising physician reviews the details of the patient's management with the student. The
physician is aware of the student's interest in critical care and asks if she would like to practice performing
� procedures, including pericardiocentesis and intubation, on the deceased patient. Which of the following is the
11
most appropriate action by the student?
12
13
Q A. Defer the procedures until a death certificate is completed

15 Q B. Defer the procedures until an autopsy is performed


16
17 Q C. Do not perform the procedures because they are unethical
18
Q D. Perform the procedures only if permission of the family is obtained
19
20 Q E. Practice the procedures, but only under direct supervision
21
22
23 Submit
24
25
26
27

Block Time Remaining: 00:43:21


28
29 TIMEDTUTOR

03: 42 �
T• l/vl l/1 •
1M
1 •
=: ltem _1 4of30
:. ? 61 � � , � 0
2 Question Id: 8957 - ?Mark <JPrevious Next
[:> Tutorial Lab Values Notes Calculator Reverse Color Text Zoom
3 •
4 A 4th-year medical student working in the intensive care unit is helping to care for a 75-year-old man with end­
5
stage lung cancer. The patient suffers a cardiac arrest and, despite intensive resuscitation and emergency
6
7
management, cannot recover and is declared dead 15 minutes later. Team members provide emotional support to
8 his family. The supervising physician reviews the details of the patient's management with the student. The
physician is aware of the student's interest in critical care and asks if she would like to practice performing
� procedures, including pericardiocentesis and intubation, on the deceased patient. Which of the following is the
11
most appropriate action by the student?
12
13
A. Defer the procedures until a death certificate is completed [0%]

15 B. Defer the procedures until an autopsy is performed [0%)


16
17 x �· C. Do not perform the procedures because they are unethical [33%]
18
19
., D. Perform the procedures only if permission of the family is obtained [60%]
20 E. Practice the procedures, but only under direct supervision [3%]

I
21
22
23
lnc�rrect
24 l1o. 60% II\ 3 Seconds i==l 09/21/2018
Corr�t answer L!!!. Answere<I correctly � TimeSpent l.!!!.I Last Update<!
25 D
26
27

Block Time Remaining: 00:43:19


28
29 TIMEDTUTOR

03: 42 �
T• l/vl l/1 •
1M
1 •
=: ltem _1 4of30
:. ? 61 � � , � 0
2 Question Id: 8957 - ?Mark <J
Previous Next
[:> Tutorial Lab Values Notes Calculator Reverse Color Text Zoom
3 •
The ethical dilemma of using newly deceased patients for training purposes involves weighing the confl icting
4
5
considerations of respect for patient integrity with the need to train health care providers to perform lifesaving
6 procedures. The consensus of major medical organizations (eg, American Medical Association, American Heart
7 Association's Emergency Cardiovascular Care Committee, Society for Academic Emergency Medicine) is that it is
8 ethical for students to perform procedures for training purposes as long as the physician obtains permission from
the family (or the patient prior to death) before the student performs the procedures. The training must occur as
� part of a structured training sequence and be performed under close supervision.
11
12 If the patient's previously expressed consent is not documented and an appropriate family member cannot be
13
found to grant permission, procedures for training purposes should not be performed. Physicians should also be

15
aware that approaching a family for permission to perform procedures shortly after a death may be overwhelming
16 and distressing.
17
(Choices A and B) It is not necessary for an autopsy or death certificate to be completed prior to performing a
18
19
training procedure.
20 (Choice C) Performing training procedures on newl y deceased patients is considered ethical if the family (or the
21
patient prior to death) has provided permission.
22
23 (Choice E) Performing any procedure requires permission of the family (or the patient prior to death). If
24
permission is obtained, the procedure should be performed under direct supervision.
25
26 Educational objective:
27 According to ethical guidelines, permission must be obtained from the family (or from the patient prior to death) •
Block Time Remaining: 00:43:19
28
29 TIMEDTUTOR

03: 42 �
T• l/vl l/1 •
1M
1 •
=: ltem _1 4of30
:. ? 61 � � , � 0
2 Question Id: 8957 - ?Mark <J Previous Next
[:> Tutorial Lab Values Notes Calculator Reverse Color Text Zoom
3 •
- - - - - • • • • • + • - - - -- · - - - - - - - - - - - - - • • • • • • • • - •

4 If the patient's previously expressed consent is not documented and an appropriate family member cannot be
5
found to grant permission, procedures for training purposes should not be performed. Physicians should also be
6
7
aware that approaching a family for permission to perform procedures shortly after a death may be overwhelming
8 and distressing.

(Choices A and B) It is not necessary for an autopsy or death certificate to be completed prior to performing a

11
training procedure.
12 (Choice C) Performing training procedures on newl y deceased patients is considered ethical if the family (or the
13
patient prior to death) has provided permission.
15 (Choice E) Performing any procedure requires permission of the family (or the patient prior to death). If
16 permission is obtained, the procedure should be performed under direct supervision.
17
18 Educational objective:
19 According to ethical guidelines, permission must be obtained from the family (or from the patient prior to death}
20
before procedures can be performed on a newly deceased patient for training purposes.
21
22
23 References
24
• Ethical issues in the emergency department: consent for procedure training on newly deceased patients.
25
26 Copyright© UWo�d. All rigllts reserved.
27 •
Block Time Remaining: 00:43:19
28
29 TIMEDTUTOR

03: 42 �
T• l/vl l/l •
1M
1 •
=: ltem _1 5of30
:. ? 61 � � , � 0
2 Question Id: 8942 - ?Mark <J
Previous Next
[:> Tutorial Lab Values Notes Calculator Reverse Color Text Zoom
3 •
4 A 48-year-old woman is admitted to the oncology service for the 4th time in 2 months. She was diagnosed with
5 breast cancer 2 years ago and has endured surgery, chemotherapy, and radiation. She also participated in
6
multiple experimental trials. Unfortunately, her disease has progressed. Today, the patient is hospitalized for
7
8
hemoptysis and shortness of breath. She complains of severe pain in her back and has lost another 5 pounds
since she was discharged 2 weeks ago. Her body mass index is 17 kg/m2. The patient has expressed a wish to


stop all treatment, go home, and enjoy her remaining days. She is found to have the capacity to make this
11 decision. The oncologist, patient, and patient's family decide that hospice care is the most appropriate plan.
12
Which of the following is considered a requirement for referring a patient for hospice care?
13
14
O A. Greater than 75% decline in ability to complete activities of daily living
16 O B. Lack of family or other caregivers to assist the patient
17
18 0 C. Patient has a do not resuscitate order
19
20
0 D. Patient has a prognosis of S6 months
21 0 E. Patient has decision-making capacity
22
23 0 F. Patient is actively dying
24
25
26 Submit
27 •
Block Time Remaining: 00:43:18
28
29 TIMEDTUTOR

03: 42 �
T• l/vl l/1 •
1M
1 •
=: ltem 1 5 of30
_ ? 61 � � , � 0
2 Question Id: 8942
:.
- ?Mark <J
Previous Next
[:> Tutorial Lab Values Notes Calculator Reverse Color Text Zoom
3 •
4 A 48-year-old woman is admitted to the oncology service for the 4th time in 2 months. She was diagnosed with
5
breast cancer 2 years ago and has endured surgery, chemotherapy, and radiation. She also participated in
6
7
multiple experimental trials. Unfortunately, her disease has progressed. Today, the patient is hospitalized for
8 hemoptysis and shortness of breath. She complains of severe pain in her back and has lost another 5 pounds
since she was discharged 2 weeks ago. Her body mass index is 17 kg/m2• The patient has expressed a wish to
� stop all treatment, go home, and enjoy her remaining days. She is found to have the capacity to make this
11
decision. The oncologist, patient, and patient's family decide that hospice care is the most appropriate plan.
12
13
Which of the following is considered a requirement for referring a patient for hospice care?

A. Greater than 75% decline in ability to complete activities of daily living [3°/o]
14

16
B. Lack of family or other caregivers to assist the patient [1%]
17
18 x,.,.....,_
- C. Patient has a do not resuscitate order [5%]
19
20
� D. Patient has a prognosis of ::56 months [80%]
21
E. Patient has decision-making capacity [4%]
22
23 F. Patient is actively dying [3%]
24
25
26
Incorrect
27 :I,.
Block Time Remaining: 00:43:15
28
29 TIMEDTUTOR

03: 42 �
T• l/vl l/1 •
1M
1 •
=: Item _1 5of30
:. ? 61 l:°f" � , � 0
2 Question Id: 8942 - ?Mark <J
Previous Next
[:> Tutorial Lab Values Notes Calculator Reverse Color Text om
Zo
3 •
4 Hospice model
5
6
• Focus on quality of life, not cure or life prolongation
7
Symptom control (pain, nausea, dyspnea, agitation, anxiety, depression)
8
• Interdisciplinary team (medical, nursing, psychosocial, spiritual, bereavement care)
• Services provided at home, assisted living facility, or dedicated facility

• Requires survival prognosis of s6 months
11
12 ©USMLEWorl d, LLC
13
14 Hospice services provide interdisciplinary palliative care for patients with a prognosis of <6 months. Hospice
care involves a patient-centered approach that emphasizes quality of life, comfort, and death with dignity. It is
16 appropriate when aggressive curative treatments are no longer beneficial or desired. The physician must
17
substantiate a prognosis of <6 months with documentation of irreversible decline in clinical and functional status.
18
19 Hospice services include comfort measures (eg, control of pain and dyspnea), logistical support (eg, home health
20 care, durable medical equipment), psychological and spiritual counseling, and bereavement support. Hospice
21
services can be provided in a home, nursing home, assisted living facility, or dedicated facility.
22
23 Common misunderstandings regarding hospice eligibility include beliefs that the patient must be actively dying and
24 have a do not resuscitate order (Choices C and F). While most patients on hospice care have clear instructions to
25
decline heroic life-sustaining measures, this is not a prerequisite to receive hospice services.
26
27 (Choice A) Although functional decline is involved in assessing prognosis, it is not a required criterion for hospice •
Block Time Remaining: 00:43:15
28
29 TIMEDTUTOR

03: 42 �
T• l/vl l/1 •
1M
1 •
=: ltem 1 5 of30
_ ? 61 � � , � 0
2 Question Id: 8942
-- - -- -- - -- --
:.
- ? Mark
--- - -
<J
Previous
-- - - [:>
Next Tutorial
-- -- - - -- -- --- -- - - - -- -- - -
Lab Values
--- --
Notes
-----
Calculator Reverse Color Text Zoom
3 •
4 services can be provided in a home, nursing home, assisted living facility, or dedicated facility.
5
Common misunderstandings regarding hospice eligibility include beliefs that the patient must be actively dying and
6
7
have a do not resuscitate order (Choices C and F). While most patients on hospice care have clear instructions to
8 decline heroic life-sustaining measures, this is not a prerequisite to receive hospice services.

(Choice A) Although functional decline is involved in assessing prognosis, it is not a required criterion for hospice

11
eligibility.
12 (Choice B) Lack of family and caregiver assistance is not a requirement. In fact, many hospice services are
13
designed to provide support for families and caregivers.
14
(Choice E) Patients should be included in all decisions as long as they have capacity. In those lacking capacity,
16 decisions regarding hospice care can be made using advanced planning directives or surrogate decision makers.
17
18 Educational objective:
19 Hospice is a palliative, interdisciplinary model of care for patients with a prognosis of <6 months. The focus is on
20
symptom control; quality of life; and psychosocial, spiritual, and bereavement care.
21
22
23 References
24
• Hospice care in the United States.
25
26 Copyright© UWo�d. All rigllts reserved.
27 •
Block Time Remaining: 00:43:15
28
29 TIMEDTUTOR

03: 42 �
T• l/vl l/l •
1M
1 �
=: ltem _1 6 of30
:. ? 61 � � , � 0
2 Question Id: 11911 - ?Mark <J
Previous Next
[:> Tutorial Lab Values Notes Calculator Reverse Color Text Zoom
3
4 A senior resident attends a medical conference at which a table sponsored by a drug company offers free fl ash
5
drives for attendees. The resident needs a new fl ash drive for presentations and asks a friend whether it is
6
7
appropriate to take one. The friend responds, "I think it is okay. It is not an expensive item and it won't affect how
8 you prescribe." According to professional ethical guidelines, which of the following is the most appropriate course
of action?

11 O A. Do not take the flash drive as it is unethical to accept any gifts from pharmaceutical companies
O B. Do not take the flash drive as only small gifts that directly benefit patients are acceptable
12
13
14
O C. Take the flash drive as the gift costs less than $50, which is acceptable
15
O D. Take the flash drive as this gift will not influence patient care
O E. Take the flash drive, but only for personal and not professional use
17
18
19
20
21 Submit
22
23
24
25
26
27

Block Time Remaining: 00:43:14


28
29 TIMEDTUTOR

03: 42 �
T• l/vl l/1 •
1M
1 •
=: ltem 1 6 of30
_ ? 61 � � , � 0
2 Question Id: 11911
:.
- ?Mark <JPrevious Next
[:> Tutorial Lab Values Notes Calculator Reverse Color Text Zoom
3 •
4 A senior resident attends a medical conference at which a table sponsored by a drug company offers free fl ash
5
drives for attendees. The resident needs a new fl ash drive for presentations and asks a friend whether it is
6
7
appropriate to take one. The friend responds, "I think it is okay. It is not an expensive item and it won't affect how
8 you prescribe." According to professional ethical guidelines, which of the following is the most appropriate course
of action?

11 A. Do not take the flash drive as it is unethical to accept any g ifts from pharmaceutical companies [32%]
12
13
B. Do not take the flash drive as only small gifts that directly benefit patients are acceptable [26%]
14
C. Take the flash drive as the gift costs less than $50, which is acceptable [22%]
15
X Q D. Take the flash drive as this gift will not influence patient care [14%]
17
18 E. Take the flash drive, but only for personal and not professional use [4%]

I
19
20
21 lnco�ect
11L!!!. 26%
.. Answered ,i'\ 3 Seconds F=l 09/26/2018
22 Corr�t answer correctly '-::J TimeSpent l.!!!.I Last Updated
23 B
24
25
Explanation
26
27
28
Block Time Remaining: 00:43:12
29 TIMEDTUTOR

03: 42 �
T• l/vl l/1 •
1M
1 •
=: ltem 1 6 of30
_ ? 61 � � , � 0
2 Question Id: 11911
:.
-[:> ?Mark <J
Previous Next Tutorial Lab Values Notes Calculator Reverse Color Text Zoom
3 •
Physicians should exercise caution when accepting gifts from external parties, such as pharmaceutical and
4
5 medical devi ce companies, that have a direct interest in a medical practice. These gifts can influence a
6 physician's prescriptions or practices, even on a subconscious level.
7
To safeguard patients and preserve the reputation of doctors, the American Medical Association (AMA) guidelines
8
(Opinion 8.061 - Gifts to Physicians from Industry) suggest accepting nonmonetary gifts from interested parties
only if the g ifts directly benefit patient care and are of small monetary value (eg, unbiased educational

11 materials, drug samples). No gifts of cash or high value, or with implied reciprocity, should be accepted. The fl ash
12 drive, although not of high monetary value (Choice C), does not directly benefit patient care and should not be
13
accepted.
14
15 (Choice A) According to AMA guidelines, nonmonetary gifts may be accepted if they directly benefit patients and
are not of high value. Some physicians have decided that accepting any gift from the pharmaceutical industry,
17
regardless of monetary value or alleged purpose, is against their personal ethics.
18
19 (Choice D) Even when physicians do not consciously believe they are being unduly influenced, a gift can affect
20
self-judgment in subtle or subconscious ways and may inadvertently cause harm to patients. Acceptance of any
21
gift should be considered with caution.
22
23 (Choice E) Even if the fl ash drive is for only personal use, it may still infl uence practice and it does not directly
24
benefit patient care. Displaying the drive may also promote pharmaceutical marketing.
25
26 Educational objective:
27 Accepting gifts from interested third parties can influence a physician's practice in subtle or subconscious ways. •
28
Block'....... ,,o;;..,........."• .......43:12
29 TIMEDTUTOR
1M
1 •
=: ltem 1 6 of30
_ ? 61 � � , � 0
2 Question Id: 11911
:.
- ?Mark <J
Previous Next
[:> Tutorial Lab Values Notes Calculator Reverse Color Text Zoom
3 •
only if the g ifts directly benefit patient care and are of small monetary value (eg, unbiased educational
4
5 materials, drug samples). No gifts of cash or high value, or with implied reciprocity, should be accepted. The fl ash
6 drive, although not of high monetary value (Choice C), does not directly benefit patient care and should not be
7 accepted.
8
(Choice A) According to AMA guidelines, nonmonetary gifts may be accepted if they directly benefit patients and
are not of high value. Some physicians have decided that accepting any gift from the pharmaceutical industry,

11 regardless of monetary value or alleged purpose, is against their personal ethics.
12
13 (Choice 0) Even when physicians do not consciously believe they are being unduly influenced, a gift can affect
14 self-judgment in subtle or subconscious ways and may inadvertently cause harm to patients. Acceptance of any
15 gift should be considered with caution.

17 (Choice E) Even if the fl ash drive is for only personal use, it may still infl uence practice and it does not directly
18 benefit patient care. Displaying the drive may also promote pharmaceutical marketing.
19
Educational objective:
20
21 Accepting gifts from interested third parties can influence a physician's practice in subtle or subconscious ways.
22 Only nonmonetary gifts that are of minimal value and that directly benefit the patient, such as unbiased educational
23 material or drug samples, should be considered.
24
25
26 References
27

Block Time Remaining: 00:43:12


28
29 TIMEDTUTOR

03: 42 �
T• l/vl l/1 •
1M
1 �
=: ltem _1 7 of30
:. ? 61 � � , � 0
2 Question Id: 4653 - ?Mark <J
Previous Next
[:> Tutorial Lab Values Notes Calculator Reverse Color Text Zoom
3
4 A 45-year-old man is brought to the emergency department following a serious motor vehicle collision on a
5
California interstate highway. Despite all attempts to save him, his condition does not improve. The physician
6
7
caring for him believes that he is brain dead. On examination, the patient's temperature is 36. 7 C (98 F ) and blood
8 pressure is 104/60 mm Hg. He is on mechanical ventilation. Neurologic examination is consistent with brain
death. Laboratory and imaging studies are not suggestive of a reversible cause. Apnea testing shows no
� spontaneous breathing. In accordance with local regulations, a second physician confirms the diagnosis of brain
11
death. The patient's family understands the prognosis. Unfortunately, the patient does not have an advanced
12
directive or durable medical power of attorney. Which of the following steps is legally necessary to remove this
13
14 patient from the ventilator?
15
16 0 A. Court-appointed legal guardianship
18
0 B. Ethics consultation
19 0 C. Medical examiner notification
20
21 0 D. No additional steps are necessary
22
23
O E. Permission from the patient's next of kin
24
25
Submit
26
27

Block Time Remaining: 00:43:11


28
29 TIMEDTUTOR

03: 42 �
T• l/vl l/1 •
1M
1 •
=: ltem _1 7 of30
:. ? 61 � � , � 0
2 Question Id: 4653 - ?Mark <JPrevious Next
[:> Tutorial Lab Values Notes Calculator Reverse Color Text Zoom
3 •
4 A 45-year-old man is brought to the emergency department following a serious motor vehicle collision on a
5
California interstate highway. Despite all attempts to save him, his condition does not improve. The physician
6
7
caring for him believes that he is brain dead. On examination, the patient's temperature is 36. 7 C (98 F ) and blood
8 pressure is 104/60 mm Hg. He is on mechanical ventilation. Neurologic examination is consistent with brain
death. Laboratory and imaging studies are not suggestive of a reversible cause. Apnea testing shows no
� spontaneous breathing. In accordance with local regulations, a second physician confirms the diagnosis of brain
11
death. The patient's family understands the prognosis. Unfortunately, the patient does not have an advanced
12
directive or durable medical power of attorney. Which of the following steps is legally necessary to remove this
13
14 patient from the ventilator?
15
16 X@ A. Court-appointed legal guardianship [2%>]

B. Ethics consultation [0%>]


18
19 C. Medical examiner notification [1 %>]
20
21 D. No additional steps are necessary [51 %>]
22
E. Permission from the patient's next of kin [43%>]
23
24
25
lnc�rect
II\ 4 Seconds
26
Iii, 51% � 10/21/2018
27 Corre<:t answer L!!!. Answere<l correctly '-::J TimeSpent l.!!!.I
Last Update<l

Block Time Remaining: 00:43:08


28
29 TIMEDTUTOR

03: 43 �
T• l/vl l/1 •
1M
1
2

=:
ltem 1 7of30
_
Question Id: 4653
:.
- ?
Mark <J
Previous Next
[:>
?
Tutorial
61
Lab Values

Notes

Calculator
,
Reverse Color
� 0
Text Zoom
3 •
4 Brain death is defined as irreversible loss of function of the whole brain, including the brainstem. There are
5
several criteria for declaring brain death. Brain death is a legally acceptable definition of death, and art ificial life
6
7
support does not need to be continued. In this patient's case, no further steps are legally required to remove him
8 from the ventilator. Some states, such as New York and New Jersey, have regulations in place in case the
declaration of legal death based on brain death is in violation of an individual's religious beliefs.

11
(Choices A and 8) A court-appointed legal guardian is sometimes needed when an individual (eg, senior citizen)
12 does not have the capacity to make medical or life decisions and there is no healthcare proxy. Involvement of the
13 hosp ital ethics committee is appropriate when the course of a patient's care is unclear, particularly when a family's
14 wishes are at odds with the goals of the healthcare providers. However, neither of these interventions is required
15
in a patient with brain death.
16
(Choice C) Patients in the hospital who die due to an unknown cause, a medical complication, suspected illegal
18 activities, or within 24 hours of presentation must generally be reported to the coroner or medical examiner . This
19
patient may need to be reported but not until after he has been disconnected from the ventilator.
20
21 (Choice E) It is important to notify this patient's family members of his condition and to discuss the removal of
22
artificial life support. However, family permission is not legally required to discontinue mechanical ventilation in a
23
patient with brain death.
24
25 Educational objective:
26
Brain death refers to a total loss of brain function and is a legally acceptable definition of death.
27 •
28
Block Time Remaining: 00:43:08
29 TIMEDTUTOR

03: 43 �
T• l/vl l/1 •
1M
1 •
=: Item 17 of 30
_ ? 61 l:°f" � , � 0
2 Question Id: 4653 :.
- ?Mark Previous
<J Next
[:> Tutorial Lab Values Notes Calculator ReverseColor ext om
T Zo
3 •
4 Exhibit Display o] �
5
Diagnosis of brain death
6
7 Clinical criteria
8 • Cli ni cal /brai n imagi ng evidence of devastating known cause
• Absence of confounding factors (eg, sedatives, metabolic)
• No evidence of drug intoxication or poisoning
• Core temperature >36 C (97 F), systolic BP >100 mm Hg
� Not all clinical
11 criteria present
Present
12 but high
suspicion of
13 Neurologic examination brain death
14 • Coma Difficult to
15 • Absent brain-or ginat ng motor response
i i
(eg, flexor & extensor posturi ng)
evaluate
or patient
'
Ancillary testing
16
• Absent oculovestibular reflexes paralyzed
(caloric response) -----------► • E ectroencepha ography
l l
18 • Absent cough with tracheal suctioning showing el ectrocerebral
19 • Absent corneal & light pupillary reflex sil ence (possibl e fal se
• Absent rooting or sucking reflexes positives/negatives) or
20 • absence of
Absent gag reflex �
21 somatosensory or
brai nstem-evoked
22 Present
23
,, potential s

24 Apneatest Difficult to
• Absent intracrania b ood
l l
fl ow on brai n imagi ng
25
• Pre-oxygenate & disconnect from venti ator
l interpret or (CT anaioaraohv/MR
26 Zoom In Reset Add To Flash Card
27 •
28
Block Time Remaining: 00:43:08
29 TIMEDTUTOR

03: 43 �
T• l/vl l/1 •
1M
1 •
=: Item _1 7 of 30
? 61 l:°f" � , � 0
2 Question Id: 4653 -
:. ?Mark <J
Previous Next
[:> Tutorial Lab Values Notes Calculator Reverse Color Text om
Zo
3 •
4
5 or patient
6 • Absent oculovestibular reflexes paralyzed Ancillary testing
(caloric response) ........... ► • E ectroencepha ography
l l
7 showing el ectrocerebral
• Absent cough with tracheal suctioning
8 • Absent corneal & light pupillary reflex sil ence (possibl e fal se
• Absent rooting or sucking reflexes positives/negatives) or
• Absent gag reflex absence of
� somatosensory or
brai nstem-evoked
11 Present potential s
12
13 Apneatest Difficult to
• Absent intracrania b ood
l l
fl ow on brai n imagi ng
14 • Pre-oxygenate & disconnect from venti ator
l interpret or (CT angiography/MR
• Brai n death confirmed by absent inconclusive angiography,
15
respi ratory response for 8 - 10 minutes
........... ► transcranial Doppl er, or
16
with PaCO2 >60 mm Hg (or >20 mm carotid angiography)
Hg above basel ine) & arterial pH <7.28

Posi tive
18
19 Positive
20
Meets local legal requirements
21
22 Yes
23

Brain death
24
25
26 Zoom In Reset Add To Flash Card
27 •
Block Time Remaining: 00:43:08
28
29 TIMEDTUTOR

03: 43 �
T• l/vl l/1 •
1M
1 •
=: ltem 1 8 of30
_ ? 61 � � , � 0
2 Question Id: 3240
:.
- ?Mark <J
Previous Next
[:> Tutorial Lab Values Notes Calculator Reverse Color Text Zoom
3 •
You are covering for a colleague who is out of town when a 65-year-old woman arrives at the clinic with her son.
4
She has experienced markedly decreased urine output for the past 24 hours. Her medical problems include type 2
5
6 diabetes mellitus, hypertension, and ischemic heart disease. The colleague performed coronary angiography on
7 the patient 5 days ago, and she was discharged home the next day in stable conditi on. Her temperature is 37° C
8 (98.6° F ), blood pressure is 140/92 mm Hg, pulse is 88/min, and respirations are 14/min. Laboratory results show
creatinine of 3.4 mg/dl and blood urea nitrogen of 40 mg/dl. Acute tubular necrosis secondary to the contrast
� used during angiography is suspected. After reviewing the notes in her medical chart before and after the
11
12 procedure, you realize that this complication could have been prevented with adequate hydration pri or to
13 angiography. The patient has provided consent for her case to be discussed with her son. He is very concerned
14 and asks why his mother is unable to urinate. Which of the following is the most appropriate response to this
15
question?
16
17
O A. I will ask the patient's regular physician to call you and discuss the current condition.

19 O B. The patient's condition may be related to her recent angiography, but I will discuss it wi th her regular
20 physician before giving a definitive answer.
O C. This complication could have been prevented if the physician had given intravenous hydration prior to
21
22
23 the procedure. Unfortunately, the physician forgot to do this.
24
25
O D. The cause of her condition is unlikely to ever be known, but she will be given excellent treatment.

26 O E. The patient is at high risk for complications due to her underlying health issues, and this is an expected
27 event that occurs frequently after angiography. •
Block Time Rem........111• .......-•..,
28
...
29 TIMEDTUTOR

03: 43 �
T• l/vl l/1 •
1M
1 •
=: ltem 1 8 of30
_ ? 61 � � , � 0
:.
?Mark <J
... . . . .
Question Id: 3240 - Previous Next Tutorial Lab Values Notes Calculator Reverse Color Text Zoom
. .... -. . .. . .... . . .
2 [:>
3 .... ..... ·· - "" "" •
°
4 (98.6 F), blood pressure is 140/92 mm Hg, pulse is 88/min, and respirations are 14/min. Laboratory results show
5 creatinine of 3.4 mg/dl and blood urea nitrogen of 40 mg/dl. Acute tubular necrosis secondary to the contrast
6
used during angiography is suspected. After reviewing the notes in her medical chart before and after the
7
procedure, you realize that this complication could have been prevented with adequate hydration prior to
8
angiography. The patient has provided consent for her case to be discussed with her son. He is very concerned
and asks why his mother is unable to urinate. Which of the following is the most appropriate response to this

11 question?
12
13 A. I will ask the patient's regular physician to call you and discuss the current condition. [4%]
14
15 � � B. The patient's condition may be related to her recent angiography, but I will discuss it wi th her regular
16 physician before giving a definitive answer. [83%]
17
C. This complication could have been prevented if the physician had given intravenous hydration pri or to
19
the procedure. Unfortunately, the physician forgot to do this. [6%>]
20 D. The cause of her condition is unlikely to ever be known, but she will be given excellent treatment. [0%]
21
22 E. The patient is at high risk for complications due to her underlying health issues, and this is an expected
23 event that occurs frequently after angiography. [5%>]
24
25
26
l1o. 83% II\ 14 Seconds i==l 08/03/2018
27 Correct L!!!. Answere<I correctly '-::.J
Time Spent l.!!!.I Last Update<! •
Block Time Remaining: 00:42:54
28
29
Windows Media Player

03: 43 �
T• l/vl l/l •
1M
1 •
=: ltem 1 8 of30
_ ? 61 � � , � 0
2 Question Id: 3240
:.
- ?Mark <J
Previous Next
[:> Tutorial Lab Values Notes Calculator Reverse Color Text Zoom
3 •
Handling the aftermath of a mistake made by a colleague can be awkward, especially when the patient's family is
4
5
pressing for detailed information. Nevertheless, it is the acting physician's obligation to discuss the essential truth
6 with the patient in a tactful and concise manner. The colleague is not available in this case. Therefore, a brief and
7 honest explanation should be provided to the patient with the understanding that a definitive answer will be
8 available once the matter has been fully discussed with the colleague. In the meantime, the patient has evidence
of acute renal injury likely due to acute tubular necrosis (ATN), and appropriate therapy should be started to treat
� her acute condition.
11
12 Physicians should consider admitting a mistake, especially under the following circumstances:
13
14 • Actual patient harm
15 • Clear or potential clinical significance
16 • An unwanted treatment, device, or substance reaching the patient
17
• An unanticipated outcome
19
• An unexpected safety event
20 (Choice A) For a physician to evade responsibility by saying he/she is not the patient's regular physician is
21
inappropriate. The acting physician is obligated to provide care that is in the patient's best interest even if it could
22
23 expose a mistake made by a colleague.
24
(Choice C) Although it is true that intravenous hydration was not given prior to the procedure, it is important to get
25
additional information from the regular physician before admitting this mistake. There might be other reasons for
26
27 not giving intravenous fluids. Patients can still incur contrast-induced kidney injury despite all precautionary

Block Time Remaining: 00:42:54
28
29 TIMEDTUTOR

03: 43 �
T• l/vl l/1 •
1M
1 •
=: ltem 1 8 of30
_ ? 61 � � , � 0
2 Question Id: 3240
:.
- ?Mark <J
Previous Next
[:> Tutorial Lab Values Notes Calculator Reverse Color Text Zoom
3 •
4 (Choice A) For a physician to evade responsibility by saying he/she is not the patient's regular physician is
5 inappropriate. The acting physician is obligated to provide care that is in the patient's best interest even if it could
6 expose a mistake made by a colleague.
7
8 (Choice C) Although it is true that intravenous hydration was not given prior to the procedure, it is important to get
additional information from the regular physician before admitting this mistake. There might be other reasons for
� not giving intravenous fluids. Patients can still incur contrast-induced kidney injury despite all precautionary
11
measures.
12
13 (Choice 0) To say that the cause of this patient's condition may never be known could be dishonest, particularl y
14 since the sequence of events very likely has an explanation. The acting physician has an obligation to tell the
15
essential truth to the patient in a tactful but honest way. However, the physician should fully discuss the matter with
16
17
the colleague before providing specific details.

(Choice E) The patient does have significant risk factors for ATN. However, telling this patient and her family that
19
acute ATN is a common complication of angiograms is dishonest. This complication may have been avoided with
20
21
adequate precautionary measures, such as hydration.
22 Educational objective:
23
A physician has a moral responsibility to act in the patient's best interests. When a physician's mistake is
24
25 discovered, the facts should be clarified and the truth told to the patient.
26 Copyright© UWo�d. All rigllts reserved.
27 •
Block Time Remaining: 00:42:54
28
29 TIMEDTUTOR

03: 44 �
T• l/vl l/l •
1M
1 •
=: ltem _1 9 of30
:. ? 61 � � , � 0
2 Question Id: 3232 - ?Mark <J
Previous Next
[:> Tutorial Lab Values Notes Calculator Reverse Color Text Zoom
3 •
4 A 65-year-old heavy smoker with a lengthy history of chronic obstructive pulmonary disease comes to the office
5
due to recent-onset hemoptysis. He reports 3-5 episodes of coughing up blood in the past month. He also reports
6
7
a 9.1-kg (20-lb) weight loss in the past 2 months. Chest radiograph reveals a suspicious lesion, and a
8 bronchoscopy followed by biopsy establishes the diagnosis of squamous cell carcinoma of the lung. When the
patient is informed of his condition and the prognosis, he asks that no one in his family be told the news yet. He
� seems tearful but cognitively intact. His wife, who usually brings him to his visits and is also his health care proxy,
11
calls the next day to inquire about the pathology report. She says she is deeply concerned because her husband
12
is reluctant to discuss his condition. Which of the following is the most appropriate course of action?
13
14
15
O A. Ask the wife to come to the office with a copy of the patient's health care proxy form.
16 O B. In response to the wife's concerns, ask her to come to the office to further review the patient's status.
17
18 O C. In response to the wife's concerns, inform her that the patient will receive comprehensive care.
20
O D. Inform the patient that his wife must eventually be told if the patient refuses to do so himself.
21 O E. Inform the wife of the pathology report results as she is his health care proxy.
22
23 O F. Inform the wife that it would not be appropriate to discuss the patient's information.
24
25
26 Submit
27 •
Block Time Remaining: 00:42:52
28
29 TIMEDTUTOR

03: 44 �
T• l/vl l/1 •
1M
1 •
=: ltem _1 9 of30
:. ? 61 � � , � 0
2 Question Id: 3232 - ?Mark <J
Previous Next
[:> Tutorial Lab Values Notes Calculator Reverse Color Text Zoom
3 •
4 A 65-year-old heavy smoker with a lengthy history of chronic obstructive pulmonary disease comes to the office
5
due to recent-onset hemoptysis. He reports 3-5 episodes of coughing up blood in the past month. He also reports
6
7
a 9.1-kg (20-lb) weight loss in the past 2 months. Chest radiograph reveals a suspicious lesion, and a
8 bronchoscopy followed by biopsy establishes the diagnosis of squamous cell carcinoma of the lung. When the
patient is informed of his condition and the prognosis, he asks that no one in his family be told the news yet. He
� seems tearful but cognitively intact. His wife, who usually brings him to his visits and is also his health care proxy,
11
calls the next day to inquire about the pathology report. She says she is deeply concerned because her husband
12
is reluctant to discuss his condition. Which of the following is the most appropriate course of action?
13
14
A. Ask the wife to come to the office with a copy of the patient' s health care proxy form. [2%]
15
16 X � B. In response to the wife's concerns, ask her to come to the office to further review the patient's status.
17
[1 %]
18
C. In response to the wife's concerns, inform her that the patient will receive comprehensive care. [7%]
20
D. Inform the patient that his wife must eventually be told if the patient refuses to do so himself. [3%]
21
22 E. Inform the wife of the pathology report results as she is his health care proxy. [2°/o]
23
24 F. Inform the wife that it would not be appropriate to discuss the patient's information. [82%)
25
26
27

Block Time Remaining: 00:42:49


28
29 TIMEDTUTOR

03: 44 �
T• l/vl l/1 •
1M
1 •
=: ltem _1 9 of30
:. ? 61 � � , � 0
2 Question Id: 3232 - ?Mark <J
Previous Next
[:> Tutorial Lab Values Notes Calculator Reverse Color Text Zoom
3 •
4 Health information cannot be shared with anyone (including family members) without the patient's permission. This
5 patient's wishes must be respected and his confidentiality strictly maintained. That said, he should be
6 strongly encouraged to discuss his diagnosis with his spouse as she will be significantly impacted by it. Moreover,
7
she may be able to provide moral support to her husband, thereby easing his burden. The physician should
8
encourage an open dialogue between the patient and his wife but should in no way coerce the patient to share the
information (Choice D).

11
(Choices A and E) The health care proxy is in effect only if the patient is deemed to have insufficient capacity to
12
13
make decisions regarding his medical treatment (eg, delirious, nonresponsive). In this case it is not in effect, as
14 there is no indication that the patient lacks capacity .
15
(Choices B and C) Without the patient's permission, it would be inappropriate to reveal any information to his wife
16
17 about his condition or care over the phone or in person.
18
Educational objective:
Medical information should be shared with family members only with the patient's permission.
20
21
22 References
23
24
• Patient and provider relationships: consent, confidentiality, and managing mistakes in integrated primary care
25 settings.
26
Copyright© UWo�d. All rigllts reserved.
27 •
Block Time Remaining: 00:42:49
28
29 TIMEDTUTOR

03: 44 �
T• l/vl l/1 •
1M
1 �
=: ltem _20of30
:. ? 61 � � , � 0
2 Question Id: 3749 - ?Mark <J
Previous Next
[:> Tutorial Lab Values Notes Calculator Reverse Color Text Zoom
3
4 A 44-year-old psychology professor with a history of chronic rheumatoid arthritis comes to the office for a follow-up
5
examination. She is currently taking prednisone and infliximab, a regimen that has successfully stabilized her
6
7
condition. She enjoys swimming 3 or 4 times a week, which helps with her strength and keeps her weight on
8 target. Temperature is 37.2 C (99 F ), blood pressure is 110/70 mm Hg, pulse is 78/min, and respirations are
16/min. The patient weighs 65 kg (143.3 lb) and is 162.5 cm (5 ft 4 in) tall. Physical examination reveals no
� significant changes. Toward the end of the visit, the patient mentions that a family friend who suffers with chronic
11
pain secondary to a spinal fusion has had a good response to acupuncture. She would like to try acupuncture
12
instead of the medications she is taking currently. Which of the following is the most appropriate response to this
13
14 patient's request?
15
16 O A. Acupuncture is definitely something we can try; alternative treatments can be very beneficial.
17
18
O B. Acupuncture should not be used as it has not demonstrated efficacy for rheumatoid arthritis.
19 O C. I am concerned that your condition will worsen rapidly if you stop your medications.
21 O D. I am not familiar with acupuncture but can refer you to a clinician experienced in the procedure.
22
23
O E. Tell me more about your interest in trying acupuncture for your condition.
24
25
Submit
26
27

Block Time Remaining: 00:42:48


28
29 TIMEDTUTOR

03: 44 �
T• l/vl l/1 •
1M
1 •
=: ltem _20of30
:. ? 61 � � , � 0
2 Question Id: 3749 - ?Mark <JPrevious Next
[:> Tutorial Lab Values Notes Calculator Reverse Color Text Zoom
3 •
4 A 44-year-old psychology professor with a history of chronic rheumatoid arthritis comes to the office for a follow-up
5
examination. She is currently taking prednisone and infliximab, a regimen that has successfully stabilized her
6
7
condition. She enjoys swimming 3 or 4 times a week, which helps with her strength and keeps her weight on
8 target. Temperature is 37.2 C (99 F ), blood pressure is 110/70 mm Hg, pulse is 78/min, and respirations are
16/min. The patient weighs 65 kg (143.3 lb) and is 162.5 cm (5 ft 4 in) tall. Physical examination reveals no
� significant changes. Toward the end of the visit, the patient mentions that a family friend who suffers with chronic
11
pain secondary to a spinal fusion has had a good response to acupuncture. She would like to try acupuncture
12
instead of the medications she is taking currently. Which of the following is the most appropriate response to this
13
14 patient's request?
15
16 A. Acupuncture is definitely something we can try; alternative treatments can be very beneficial. [2%>]
17
X@ B. Acupuncture should not be used as it has not demonstrated efficacy for rheumatoid arthritis. [0%]
18
19 C. I am concerned that your condition will worsen rapidly if you stop your medications. [5%>]

21 D. I am not familiar with acupuncture but can refer you to a clinician experienced in the procedure. [4%)
22
E. Tell me more about your interest in trying acupuncture for your condition. [86%]
23
24
25
lnco i:rect
II\ 4 Seconds
26
(1, , 86% � 09/11/2018
27 Correct answer L!!!. Answere<l correctly '-::J Time Spent l.!!!.I
Last Update<l
28
Block Time Remaining: 00:42:45
29 TIMEDTUTOR

03: 44 �
T• l/vl l/1 •
1M
1 •
=: ltem _20of30
:. ? 61 � � , � 0
2 Question Id: 3749 - ?Mark <J
Previous Next
[:> Tutorial Lab Values Notes Calculator Reverse Color Text Zoom
3 Patients are increasingly interested in complementary and alternative medicine (CAM), and physicians must be •
4
prepared to discuss CAM modalities with them and integrate treatments that have demonstrated safety and
5
6
efficacy. The initial response to a patient who requests an alternative approach is to explore the reasons for
7 requesting a different treatment.
8
This patient may be dissatisfied or have concerns about the quality of current care. She may be experiencing
bothersome medication side effects of which the physician is unaware. Once the patient's reasons are clarified,

11 the physician can discuss the risks and benefits of the proposed CAM treatment in addition to suggesting
12 modifications to the current treatment regimen that may address her concerns.
13
14
(Choice A) This response enthusiastically endorses the patient's request for acupuncture without first
15 understanding what is prompting her request. In addition, it makes a general statement about the benefits of CAM
16 that is inaccurate in her case given the lack of evidence for acupuncture in rheumatoid arthritis and the risks of
17 discontinuing the current treatments.
18
19 (Choice B) Although studies have not demonstrated acupuncture's efficacy for rheumatoid arthritis, this response
rejects the patient's suggestion outright without first exploring the reasons for her request. The risks and benefits
21
of the proposed treatment can be discussed after the physician has gained a better understanding of the patient's
22
perspective.
23
24 (Choice C) Warning the patient about the risks of stopping her current treatment regimen is dismissive of her
25
request and likely to damage the physician-patient relationship. The first step is to understand why she is
26
requesting a change in her treatment. The risks of stopping her current medications can be addressed at a later
27 •
Block Time Remaining: 00:42:45
28
29 TIMEDTUTOR

03: 44 �
T• l/vl l/1 •
1M
1 �
=: ltem _20of30
:. ? 61 � � , � 0
Question Id: 3749 - ?Mark <J
Previous Next Tutorial Lab Values Notes Calculator Reverse Color Text Zoom
. . [:>
.. . ' ,.
2
3 g p p g q g .
4 that is inaccurate in her case given the lack of evidence for acupuncture in rheumatoid arthritis and the risks of
5
discontinuing the current treatments.
6
7 (Choice B) Although studies have not demonstrated acupuncture's efficacy for rheumatoid arthritis, this response
8 rejects the patient's suggestion outright without first exploring the reasons for her request. The risks and benefits
of the proposed treatment can be discussed after the physician has gained a better understanding of the patient's
� perspective.
11
12 (Choice C) Warning the patient about the risks of stopping her current treatment regimen is dismissive of her
13
request and likely to damage the physician-patient relationship. The first step is to understand why she is
14
15
requesting a change in her treatment. The risks of stopping her current medications can be addressed at a later
16 point.
17
(Choice D) It would be premature to refer the patient for acupuncture without first understanding the reasons
18
19
behind her request.

Educational objective:
21
The initial response to patients suggesting complementary and alternative medicine is to explore their reasons for
22
23 requesting such treatment. Physicians must be prepared to discuss the ri sks and benefits of complementary and
24 alternative approaches to treatment and be open to integrating modalities with proven safety and efficacy.
25
26
References
27 •
Block Time Remaining: 00:42:45
28
29 TIMEDTUTOR

03: 44 �
T• l/vl l/1 •
1M
1 •
=: ltem _21 of30
:. ? 61 � � , � 0
2 Question Id: 3615 - ?Mark <J
Previous Next
[:> Tutorial Lab Values Notes Calculator Reverse Color Text Zoom
3 •
A 43-year-old patient asks the physician to examine his 75-year-old mother, who has been complaining of
4
5 headaches. When the woman presents for her first appointment, she admits to having a severe, persistent
6 headache for the past 4 weeks. She says the headache is worse in the morning and always lingers throughout the
7 day. She usually feels nauseated but has no vomiting. Her medical history is significant for severe depression and
8
anxiety, and her current medications include an antidepressant. A complete workup shows that she has a brain
tumor, likely a glioblastoma. Both she and her son (also the physician's patient) are present at today's appointment

11
to learn of the diagnosis. Before the physician has a chance to discuss the findings, the son steps outside the
12 patient's room and requests that the physician not reveal the diagnosis to his mother. Which of the following is the
13 most appropriate response?
14
15 O A. "As a physician, I always tell a patient about a serious diagnosis, as it is my moral responsibility. "
16
17 O 8. "If you feel this strongly about it, how about I delay giving her information now, but tell her at a later
18 date?"
19
20
O C. "Is there a specific reason why you do not want your mother to know the diagnosis?"
O D. "Leave the matter to me as I will determine whether she should know her diagnosis."
22
23 O E. "This news would only be upsetting to her and therefore I will not discuss the diagnosis at all."
24
25
26 Submit
27 •
Block Time Remaining: 00:42:44
28
29 TIMEDTUTOR

03: 44 �
T• l/vl l/1 •
1M
1 �
=: ltem _21 of30
:. ? 61 � � , � 0
2 Question Id: 3615 - ?Mark <J
Previous Next
[:> Tutorial Lab Values Notes Calculator Reverse Color Text Zoom
3
4 A 43-year-old patient asks the physician to examine his 75-year-old mother, who has been complaining of
5
headaches. When the woman presents for her first appointment, she admits to having a severe, persistent
6
7
headache for the past 4 weeks. She says the headache is worse in the morning and always lingers throughout the
8 day. She usually feels nauseated but has no vomiting. Her medical history is significant for severe depression and
anxiety, and her current medications include an antidepressant. A complete workup shows that she has a brain
� tumor, likely a glioblastoma. Both she and her son (also the physician's patient) are present at today's appointment
11
to learn of the diagnosis. Before the physician has a chance to discuss the findings, the son steps outside the
12
patient's room and requests that the physician not reveal the diagnosis to his mother. Which of the following is the
13
14 most appropriate response?
15
16 A. "As a physician, I always tell a patient about a serious diagnosis, as it is my moral responsibility. " [2%]
17
B. "If you feel this strongly about it, how about I delay giving her information now, but tell her at a later
18
19
date?" [0%]
20 � r C. "Is there a specific reason why you do not want your mother to know the diagnosis?" [96%]

22 D. "Leave the matter to me as I will determine whether she should know her diagnosis." [0%]
23
E. "This news would only be upsetting to her and therefore I will not discuss the diagnosis at all." [0%]
24
25
26
27
n� - • • I • I:
Block Time Remaining: 00:42:41
28
29 TIMEDTUTOR

03: 44 �
T• l/vl l/1 •
1M
1 •
=: ltem _21 of30
:. ? 61 � � , � 0
2 Question Id: 3615 - ?Mark <J
Previous Next
[:> Tutorial Lab Values Notes Calculator Reverse Color Text Zoom
3 •
In general, a patient has the right to know a diagnosis. If family members ask for information to be withheld, it is
4
imperative for the physician to understand their reasoning. Occasionally, it is in the patient's best interest to
5
6 withhold especially distressing news (eg, if a severely depressed patient might become suicidal). Therefore, it is
7 best to clarify the situation with the concerned family members first before deciding how best to proceed.
8
(Choice A) It is indeed accurate (albeit with some qualifications) for the physician to tell the patient's son of !lm
moral obligation to inform the patient of her diagnosis. However, it is better to first determine the family member's

11 reasoning for wi thholding the diagnosis before deciding how to handle the s ituation.
12
(Choice B) Immediately agreeing to withhold or delay news of the diagnosis is inappropriate as the family
13
14 member's reasoning should first be explored and discussed.
15
(Choice D) Answering the family member's request with a rude, abrupt response is unprofessional. It is important
16
17
to provide support to both the mother and her son.
18 (Choice E) While the physician may conclude that the news would be upsetting, it would be most appropriate to
19
hear the son's concerns before proceeding. For instance, the patient may be relieved to know that the etiology of
20
her headaches has been discovered despite receiving the diagnosis of cancer.
22 Educational objective:
23
Patients have the right to know their diagnoses. If family members request that the diagnosis not be revealed to
24
25 the patient, the underl ying reasons should be explored before deciding how to proceed.
26 Copyright© UWo�d. All rigllts reserved.
27 •
Block Time Remaining: 00:42:41
28
29 TIMEDTUTOR

03: 44 �
T• l/vl l/l •
1M
1 •
=: ltem _22of30
:. ? 61 � � , � 0
2 Question Id: 13465 - ?Mark <J
Previous Next
[:> Tutorial Lab Values Notes Calculator Reverse Color Text Zoom
3 •
4 An 81-year-old woman comes to the office for follow-up of type 2 diabetes mellitus, which has been well controlled
5
on a stable medication regimen for several years. Medical history is notable for hypertension and osteoporosis.
6
7
Six months ago, she suffered a nondisplaced orbital fracture. She is widowed and has lived with her daughter and
8 son-in-law for the last year. She claims that she is independent and "takes care" of herself. Blood pressure is
176/102 mm Hg and pulse is 84/min. Examination shows bruising at the right forearm and right flank. No cardiac
� murmurs are heard. Abdominal examination is unremarkable. Gait is steady and the patient does well on the
11
Timed Up and Go test. Laboratory results show a fasting glucose of 220 mg/dL and hemoglobin A1c of 9%. F our
12
months ago, glucose was 105 mg/dL and hemoglobin A1c was 7.2%. On further discussion, the patient says, "The
13
14 pharmacy charges me too much these days. I just can't afford my medication anymore." Which of the following is
15 the most likely explanation for this patient's clinical findings?
16
17 0 A. Alcohol abuse
18
19 0 B. Early-stage dementia
20
21
0 C. Elder abuse
0 D. Late-life depression
23
24 0 E. Nutritional deficiency
25
26
0 F. Psychotic disorder
27 •
Block Time Remaining: 00:42:40
28
29 TIMEDTUTOR

03: 44 �
T• l/vl l/1 •
1M
1 �
=: ltem _22of30
:. ? 61 � � , � 0
?Mark <J
2
3 · - -·-
Question Id: 13465 -
-· - · - . -
Previous
. Next
[:>
- , - _ ,_ _ • -· - .,
Tutorial Lab Values
. ... . - . ,.- - . - · · -··· · Notes Calculator Reverse Color Text Zoom

4 Six months ago, she suffered a nondisplaced orbital fracture. She is widowed and has lived wi th her daughter and
5 son-in-law for the last year. She claims that she is independent and "takes care" of herself. Blood pressure is
6
176/102 mm Hg and pulse is 84/min. Examination shows bruising at the right forearm and right flank. No cardiac
7
8 murmurs are heard. Abdominal examination is unremarkable. Gait is steady and the patient does well on the
Timed Up and Go test. Laboratory results show a fasting glucose of 220 mg/dL and hemoglobin A1c of 9%. F our

months ago, glucose was 105 mg/dL and hemoglobin A1c was 7.2%. On further discussion, the patient says, "The
11 pharmacy charges me too much these days. I just can't afford my medication anymore." Which of the following is
12
the most likely explanation for this patient's clinical findings?
13
14
A. Alcohol abuse [0%)
15
16 B. Early-stage dementia [4%)
17
18 � C. Elder abuse [90%)

X r D. Late-life depression [1%)


19
20
21 E. Nutriti onal deficiency [1 %)

23 F. Psychotic disorder [0%>)


24
25
26
lncohfect (1,, 90%
27
L!!!. Answered correctl II • •I
Block Time Remaining: 00:42:36
28
29 TIMEDTUTOR

03: 44 �
T• l/vl l/1 •
1M
1
2

= Item 22 of 30
- Question Id: 13465
_
� ?
Mark <J
Previous
C>
Next
?
Tutorial
61
Lab Values

N�tes

Calculator
,
Reverse Color
� 0
Text zoom
3 •
4 Elder abuse
5
• Female
6
7
• Dementia, chronic mental illness
8 • Functional impairments
Risk factors
• Social isolation
� • Shared living environment
11
• Poor socioeconomic status/financial stress
12
13 Physical & sexual abuse
14
• Atypical abrasions, lacerations, contusions, fractures
15
• Pain not consistent with reported etiology
16
17 • Anogenital injuries
18 • Newl y acquired STI
19
20 Psychological & verbal abuse
21 • Change in behavior/personality
• Depression/anxiety
23 Manifestations of abuse 1----------------------1
24 Neglect
25 • Inadequate nutrition or hydration
26 • Pressure ulcers
27 •
Deterioration in comorbid conditions
Block Time Remaining: 00:42:36
28
29 TIMEDTUTOR

03: 44 �
T• l/vl l/1 •
1M
1 •
=: ltem _22of30
:. ? 61 � � , � 0
2 Question Id: 13465 - ?Mark <J
Previous Next
[:> Tutorial Lab Values Notes Calculator Reverse Color Text Zoom
3 •
4 Physical & sexual abuse
5 • Atypical abrasions, lacerations, contusions, fractures
6
• Pain not consistent with reported etiology
7
• Anogenital injuries
8
• Newl y acquired STI

� Psychological & verbal abuse


11
• Change in behavior/personality
12
• Depression/anxiety
Manifestations of abuse 1----------------------1
13
14
Neglect
15
16
• Inadequate nutrition or hydration
17 • Pressure ulcers
18 • Deterioration in comorbid conditions
19
20 Financial exploitation
21 • Failure to adhere to medication regimen
• Multiple missed appointments
23
• Unpaid expenses or rent payments
24
25 STI = sexually transmitted infection.
26
27 This patient has findings that raise concern for elder abuse, including multiple injuri es and an acute change in •
Block Time Remaining: 00:42:36
28
29 TIMEDTUTOR

03: 44 �
T• l/vl l/1 •
1M
1 •
=: ltem _22of30
:. ? 61 � � , � 0
2 Question Id: 13465 - ?Mark <J
Previous Next
[:> Tutorial Lab Values Notes Calculator Reverse Color Text Zoom
3 This patient has findings that raise concern for elder abuse, including multiple injuri es and an acute change in •
4
financial status (ie, discontinuing longstanding medications due to cost), possibly due to diversion of finances by
5
6 family members. Abuse is most common in older women, especially those who are socially isolated or have a
7 s hared living environment (ie, live with multiple nonspousal others). Dementia and chronic mental illness also
8 increase the risk.

Manifestations of physical abuse in the elderl y are similar to those in younger patients, and may include

11 unexplained pain, injuries in multiple stages of healing, or injuries in unusual locations. Although osteoporosis
12 increases the risk of fracture with minor trauma, orbital fractures (as in this patient) are atypical. Elderl y
13 individuals are also at risk for fi nancial exploitation, psychological abuse, sexual abuse, and neglect. Most
14
jurisdictions require that suspected abuse be reported to adult protective services or other governmental agency.
15
16 (Choice A) Late-onset alcohol abuse is more common in elders who live alone. Living with multiple family
17
members increases the risk for elder abuse. Elder abuse is also more likely to explain this patient's injury pattern
18
(orbital fracture, multiple bruises in unusual locations).
19
20 (Choices B , D, and F) Dementia, affective disorders, and psychotic disorders are common in elderly patients and
21
can lead to nonadherence to medical therapy. However, this patient has experienced multiple injuries that suggest
abuse.
23
24 (Choice E) Nutritional deficiencies are usually seen in elders who live alone and do not have outside nutriti onal
25
support. Although a variety of vitamin deficiencies can cause bruising, this patient's orbital fracture is more
26
27
suspicious for abuse.

Block Time Remaining: 00:42:36
28
29 TIMEDTUTOR

03:45�
T• l/vl l/1 •
1M
1 �
=: ltem _22of30
:. ? 61 � � , � 0
2 Question Id: 13465 - ?Mark <J
Previous Next
[:> Tutorial Lab Values Notes Calculator Reverse Color Text Zoom
3 J • q p • • • p • • p g g y
4
5
(Choice A) Late-onset alcohol abuse is more common in elders who live alone. Living with multiple family
6 members increases the risk for elder abuse. Elder abuse is also more likely to explain this patient's injury pattern
7 (orbital fracture, multiple bruises in unusual locations).
8
(Choices B , D, and F) Dementia, affective disorders, and psychotic disorders are common in elderly patients and
can lead to nonadherence to medical therapy. However, this patient has experienced multiple injuries that suggest

11 abuse.
12
13
(Choice E) Nutritional deficiencies are usually seen in elders who live alone and do not have outside nutritional
14 support. Although a variety of v itamin deficiencies can cause bruising, this patient's orbital fracture is more
15 suspicious for abuse.
16
17 Educational objective:
18 Manifestations of physical abuse in the elderl y may include unexplained pain, injuries in multiple stages of healing,
19 or injuries in unusual locations. Elderl y individuals are also at ri sk for financial exploitation, psychological abuse,
20
sexual abuse, and neglect.
21

23 References
24
• Elder abuse.
25
26 Copyright© UWo�d. All rigllts reserved.
27 •
Block Time Remaining: 00:42:36
28
29 TIMEDTUTOR

03:45�
T• l/vl l/l •
1M
1 �
=: ltem _23of30
:. ? 61 � � , � 0
2 Question Id: 3238 - ?Mark <J
Previous Next
[:> Tutorial Lab Values Notes Calculator Reverse Color Text Zoom
3
4 A pediatrician in a group practice comes to a colleague to inquire about a patient that the colleague examined
5
earlier that morning. The pediatrician says that he recognized the patient as a former neighbor who is also a good
6
7
friend of the family. The patient was recently diagnosed as HIV positive and is now on antiretroviral medication.
8 The pediatrician asks why the patient is being treated. What is the most appropriate response to such an inquiry?


O A. Actually, we've not been able to diagnose his condition.
11
12
0 B. He is on antiretrovirals because he is HIV positive.
13 0 C. I think you should know he has an infectious disease.
14
15 0 D. It would be best for the patient to tell you the diagnosis.
16
17
0 E. It would be inappropriate for me to discuss this with you.
18
19
Submit
20
21
22

24
25
26
27

Block Time Remaining: 00:42:35


28
29 TIMEDTUTOR

03:45�
T• l/vl l/1 •
1M
1 •
=: ltem _23of30
:. ? 61 � � , � 0
2 Question Id: 3238 - ?Mark <JPrevious Next
[:> Tutorial Lab Values Notes Calculator Reverse Color Text Zoom
3 •
4 A pediatrician in a group practice comes to a colleague to inquire about a patient that the colleague examined
5
earlier that morning. The pediatrician says that he recognized the patient as a former neighbor who is also a good
6
7
friend of the family. The patient was recently diagnosed as HIV positive and is now on antiretroviral medication.
8 The pediatrician asks why the patient is being treated. What is the most appropriate response to such an inquiry?

X@ A. Actually, we've not been able to diagnose his condition. [0%>]



11
B. He is on antiretrovirals because he is HIV positive. [0%]
12
13 C. I think you should know he has an infectious disease. [0%]
14
15 D. It would be best for the patient to tell you the diagnosis. [8%]
16
E. It would be inappropriate for me to discuss this with you. [90%]
17

I
18
19
lnc�ect
II\ 5 Seconds
20
1 1 .. 90% F==I 06/03/2018
21 Corr�t answer L!!!. Answered correctly "-::.J TimeSpent l.!::.I Last Updated
E
22

24 Explanation
25
26
27
Confidentiality is one of the key components of a well-run and ethical medical practice. The obligation to maintain

Block Time Remaining: 00:42:31


28
29 TIMEDTUTOR

03:45�
T• l/vl l/1 •
1M
1 •
=: ltem _23of30
:. ? 61 � � , � 0
2 Question Id: 3238 - ?Mark <J
Previous Next
[:> Tutorial Lab Values Notes Calculator Reverse Color Text Zoom
3 Confidentiality is one of the key components of a well-run and ethical medical practice. The obligation to maintain •
4
confidentiality prohibits the physician from disclosing information about the patient's diagnosis or treatment to
5
6
anyone not directly involved in, or necessary to, the patient's management. Physicians should avoid discussing a
7 patient's medical condition in public areas where comments might be overheard.
8
In this case, although the person inquiring is a physician and colleague, he is not involved in the patient's care and
is therefore not enti tled to details regarding that care. Moreover, his inquiry is based on personal and not

11 professional curiosity. Therefore, the physician should tell the colleague that it would not be appropriate to discuss
12 the patient's condition. Every effort should be made to strictly maintain the patient's confidentiality.
13
14
(Choice A) The physician should not be dishonest. Instead, the physician should simply respond that it would not
15 be appropriate to discuss the patient's diagnosis or treatment.
16
(Choice B) Divulging the diagnosis would be inappropriate as it would be a breach of confidentiality.
17
18 (Choice C) Stating that the patient has an infectious disease without further disclosure would also vi olate the
19
patient's confidentiality and is not appropriate.
20
21 (Choice D) Although it would be best for the physician's colleague to talk directly to the patient to learn of the
22 diagnosis, it is inappropriate to place the patient in a difficult situation should the colleague proceed with this
suggestion.
24
25 Educational objective:
26 Confidential patient information should be disclosed only to fellow health care workers who are directly involved in
27
the Ph sicians should avoid discussin a atient's medical condition in ublic areas where
Block Time Remaining: 00:42:31
28
29 TIMEDTUTOR

03:45�
T• l/vl l/1 •
1M
1 �
=: ltem _23of30
:. ? 61 � � , � 0
2 Question Id: 3238 - ?Mark <J
Previous Next
[:> Tutorial Lab Values Notes Calculator Reverse Color Text Zoom
3 p • • p • g • •
4
5
In this case, although the person inquiring is a physician and colleague, he is not involved in the patient's care and
6 is therefore not enti tled to details regarding that care. Moreover, his inquiry is based on personal and not
7 professional curiosity. Therefore, the physician should tell the colleague that it would not be appropriate to discuss
8 the patient's condition. Every effort should be made to strictly maintain the patient's confidentiality.

(Choice A) The physician should not be dishonest. Instead, the physician should simply respond that it would not

11 be appropriate to discuss the patient's diagnosis or treatment.
12
13
(Choice B) Divulging the diagnosis would be inappropriate as it would be a breach of confidentiality.
14 (Choice C) Stating that the patient has an infectious disease without further disclosure would also vi olate the
15
patient's confidentiality and is not appropriate.
16
17 (Choice D) Although it would be best for the physician's colleague to talk directly to the patient to learn of the
18 diagnosis, it is inappropriate to place the patient in a difficult situation should the colleague proceed with this
19
suggestion.
20
21 Educational objective:
22
Confidential patient information should be disclosed only to fellow health care workers who are directly involved in
the patient's care. Physicians should avoid discussing a patient's medical condition in public areas where
24
25 comments might be overheard. Inappropriate inquiries from colleagues curious about a patient' s medical condition
26 should be politely but firmly rebuffed.
27
II!,;I (::i II '
Block Time Remaining: 00:42:31
28
29 TIMEDTUTOR

03:45�
T• l/vl l/1 •
1M
1 �
=: ltem 24of30
_ ? 61 � � , � 0
2 Question Id: 3617
:.
- ?Mark <J
Previous Next
[:> Tutorial Lab Values Notes Calculator Reverse Color Text Zoom
3
4 A SO-year-old man comes to the physician complaining of weight loss and fatigue. A review of systems is pos itive
5
for a change in bowel habits. Further evaluation shows the presence of colon carcinoma. The patient is told the
6
7
diagnosis and is educated about his treatment options and prognosis; he listens politely and does not ask any
8 questions. He is a high school graduate and worked in a car factory until taking extended sick leave in recent
weeks. The patient appears to comprehend the information without difficulty. He says he does not want treatment
� or intervention of any kind. The physician also serves as primary care physician for the patient's wi fe. What would
11
be the most appropriate next step in addressing this situation?
12
13
Q A. Ask the patient why he does not want treatment or intervention of any kind
14
15 Q B. Contact his wife and ask her to convince him to receive treatment
16
17 Q C. Provide the patient with literature about the benefits of surgery and chemotherapy
18
Q D. Respect the patient's decision and do not schedule additional appointments
19
20 Q E. Treat the patient against his wishes , obtaining a court order if necessary
21
22
23 Submit

25
26
27

Block Time Remaining: 00:42:30


28
29 TIMEDTUTOR

03:45�
T• l/vl l/1 •
1M
1 •
=: ltem _24of30
:. ? 61 � � , � 0
2 Question Id: 3617 - ?Mark <JPrevious Next
[:> Tutorial Lab Values Notes Calculator Reverse Color Text Zoom
3 •
4 A SO-year-old man comes to the physician complaining of weight loss and fatigue. A review of systems is pos itive
5
for a change in bowel habits. Further evaluation shows the presence of colon carcinoma. The patient is told the
6
7
diagnosis and is educated about his treatment options and prognosis; he listens politely and does not ask any
8 questions. He is a high school graduate and worked in a car factory until taking extended sick leave in recent
weeks. The patient appears to comprehend the information without difficulty. He says he does not want treatment
� or intervention of any kind. The physician also serves as primary care physician for the patient's wi fe. What would
11
be the most appropriate next step in addressing this situation?
12
13
�@ A. Ask the patient why he does not want treatment or intervention of any kind [92%]
14
15 B. Contact his wife and ask her to convince him to receive treatment [0%]
16
17 C. Provide the patient with literature about the benefits of surgery and chemotherapy [3%]
18
D. Respect the patient's decision and do not schedule additional appointments [3%]
19
20 E. Treat the patient against his wishes , obtaining a court order if necessary [0%>]

I
21
22
23

Correct� (1, , 92% II\ 4 Seconds I==! 05/21/2018


25
L!!!. Answere<I correctly "-.::; Time Spent l.!!!.I Last Update<!

26
27
28
Block Time Remaining: 00:42:27
29 TIMEDTUTOR

03:45�
T• l/vl l/1 •
1M
1 •
=: ltem 24of30
_ ? 61 � � , � 0
2 Question Id: 3617
:.
- ?Mark <J
Previous Next
[:> Tutorial Lab Values Notes Calculator Reverse Color Text Zoom
3 Although this patient likely has the capacity to refuse treatment, his reasoning should first be thoroughly explored •
4
and discussed with the physician. He may be in a state of disbelief or denial or afraid of pain or unfamiliar
5
6 procedures. Patient concerns may sometimes have a more philosophical basis, such as controlling one's destiny,
7 or may be the result of inadequate understanding. For example, this patient may have known others with cancer in
8 the past and not be aware of significant advances in prolonging quality and quantity of life. Patients may also have
mental health issues such as depression that can diminish their desire for treatment. If the patient continues to
� refuse medical intervention despite clarification and reassurance from the physician, then this decision should be
11
12
respected and treatment withheld.
13 (Choice B) This patient appears to have the capacity to make informed decisions about his care. Therefore, it
14
would be inappropriate for the physician to contact family members as a means of pressuring him into compliance.
15
16 (Choice C) After the matter has been discussed with the patient, he should be given add itional literature and
17
verbal information about his disease, its prognosis, and the risks and benefits of the available treatment options.
18
This would not be the first step, however, as the physician has already provided initi al information.
19
20 (Choice D) The patient's decision should indeed be respected; however, a cancer patient is in need of ongoing
21
support. Additional educational efforts and further development of a treatment alliance, family involvement, or
22
change in the patient's condition may lead the patient to reconsider refusing treatment.
23
(Choice E) Treating this patient against his wishes would be unethical and would invi te criminal charges of
25
battery. It is therefore not an advisable course of action.
26
27 Educational objective: •
Block Time Remaining: 00:42:27
28
29 TIMEDTUTOR

03:45�
T• l/vl l/1 •
1M
1 •
=: ltem 24of30
_ ? 61 � � , � 0
2 Question Id: 3617
:.
- ?Mark <J
Previous Next
[:> Tutorial Lab Values Notes Calculator Reverse Color Text Zoom
3 •
the past and not be aware of significant advances in prolonging quality and quantity of life. Patients may also have
4
mental health issues such as depression that can diminish their desire for treatment. If the patient continues to
5
6 refuse medical intervention despite clarification and reassurance from the physician, then this decision should be
7 respected and treatment withheld.
8
(Choice B) This patient appears to have the capacity to make informed decisions about his care. Therefore, it
would be inappropriate for the physician to contact family members as a means of pressuring him into compliance.

11
(Choice C) After the matter has been discussed with the patient, he should be given add itional literature and
12
verbal information about his disease, its prognosis, and the risks and benefits of the available treatment options.
13
14 This would not be the first step, however, as the physician has already provided initi al information.
15
(Choice D) The patient's decision should indeed be respected; however, a cancer patient is in need of ongoing
16
17
support. Additional educational efforts and further development of a treatment alliance, family involvement, or
18 change in the patient's condition may lead the patient to reconsider refusing treatment.
19
(Choice E) Treating this patient against his wishes would be unethical and would invi te criminal charges of
20
21
battery. It is therefore not an advisable course of action.
22 Educational objective:
23
When a patient refuses potentially life-saving treatment, it is important to fully discuss the specific reasons for the
25 decision before honoring it.
26 Copyright© UWo�d. All rigllts reserved.
27 •
Block Time Remaining: 00:42:27
28
29 TIMEDTUTOR

03:45�
T• l/vl l/l •
1M
1 •
=: ltem 25of30
_ ? 61 � � , � 0
2 Question Id: 3616
:.
- ?Mark <J
Previous Next
[:> Tutorial Lab Values Notes Calculator Reverse Color Text Zoom
3 •
4 An unconscious 42-year-old man is brought to the emergency department by ambulance after sustaining a gunshot
5
wound to the chest during an attempted robbery at a convenience store. He is bleeding profusely. The patient is
6
7
minimally responsive to pain and is unable to provide any history. The ambulance record indicates that his initial
8 blood pressure was 120/70 mm Hg, pulse was 95/min, and respirations were 16/min. The patient's current blood
pressure is 70/40 mm Hg, pulse is 130/min, and respirations are 28/min. He has an open gunshot wound in the
� chest and an exit wound in the back. Intravenous fluids and vasopressors are started, and a blood transfusion is
11
ordered. The patient's fi ance arrives and states that he should not receive a blood transfusion because he is a
12
devout Jehovah's Witness. No evidence is found in the medical chart of advance directives or a living will
13
14 documenting the patient's wishes, and examination of his belongings show no blood refusal card. Which of the
15 following is the most appropriate next step in management of this patient?
16
17 O A. Administer high-dose erythropoietin and intravenous iron to stabilize blood loss
O B. Continue intravenous fluids and consult the hospital ethics committee
18
19
20
21
O C. Obtain a court order for approval to administer the blood transfusion
22 0 D. Proceed with the blood transfusion
O E. Withhold the blood transfusion because of the patient's religious beliefs
23
24

26
27 Submit

Block Time Remaining: 00:42:25


28
29 TIMEDTUTOR

03:45�
T• l/vl l/1 •
1M
1 •
=: ltem _25of30
:. ? 61 � � , � 0
2 Question Id: 3616 - ?Mark <J
Previous Next
[:> Tutorial Lab Values Notes Calculator Reverse Color Text Zoom
3 •
4 An unconscious 42-year-old man is brought to the emergency department by ambulance after sustaining a gunshot
5
wound to the chest during an attempted robbery at a convenience store. He is bleeding profusely. The patient is
6
7
minimally responsive to pain and is unable to provide any history. The ambulance record indicates that his initial
8 blood pressure was 120/70 mm Hg, pulse was 95/min, and respirations were 16/min. The patient's current blood
pressure is 70/40 mm Hg, pulse is 130/min, and respirations are 28/min. He has an open gunshot wound in the
� chest and an exit wound in the back. Intravenous fluids and vasopressors are started, and a blood transfusion is
11
ordered. The patient's fi ance arrives and states that he should not receive a blood transfusion because he is a
12
devout Jehovah's Witness. No evidence is found in the medical chart of advance directives or a living will
13
14 documenting the patient's wishes, and examination of his belongings show no blood refusal card. Which of the
15 following is the most appropriate next step in management of this patient?
16
17 A. Administer high-dose erythropoietin and intravenous iron to stabilize blood loss [2%]

X r B. Continue intravenous fluids and consult the hospital ethics committee [3%>]
18
19
20
C. Obtain a court order for approval to administer the blood transfusion [0%)
21
22 D. Proceed with the blood transfusion [87%]
23
24 E. Wi thhold the blood transfusion because of the patient's religious beliefs [6%>]

26
27
- I � I:
Block Time Remaining: 00:42:22
28
29 TIMEDTUTOR

03:45�
T• l/vl l/1 •
1M
1 •
=: ltem _25of30
:. ? 61 � � , � 0
2 Question Id: 3616 - ?Mark <J
Previous Next
[:> Tutorial Lab Values Notes Calculator Reverse Color Text Zoom
3 This patient is a presumed Jehovah's Witness who presents with hemorrhagic shock and is likely to benefit from •
4
a transfusion that could prevent exsanguination. Because the patient lacks decision-making capacity, providers
5
6
should determine if he has completed an advance directive that can guide treatment decisions. Most Jehovah's
7 Wi tnesses carry advance directive cards that explicitly document the individual's refusal of a blood transfusion. In
8 the case of adult patients, courts have consistently supported the right to refuse blood on religious grounds.

In this case, no advance directive card is provided and delay in treatment could place the patient in serious

11 harm. The Health Care Consent Act has been applied to Jehovah's Witnesses to allow transfusion to preserve a
12 patient's life in an emergency when no blood refusal card is present. The patient's fiance is not a legal next of kin
13 and cannot serve as a substituted decision-maker (Choice E).
14
15 (Choice A) Case reports have shown some success in using high-dose erythropoietin, crystalloids, intravenous
16 iron, and other products to address massive hemorrhage without a blood transfusion. These treatments could be
17 discussed if the patient was conscious and refused a blood transfusion.
18
19 (Choice B) Ethics committees provide advice if disagreements or ambiguity exist between health care providers
20 and a patient/patient' s family on decisions involving withholding or wi thdrawing life-sustaining treatments. This
21
patient needs treatment quickly, with no time for discussion with the ethics committee.
22
23 (Choice C) Court approval is not needed for a physician to proceed with emergency life-saving treatment for an
24 unconscious or unresponsive patient.

26 Educational objective:
27 In the absence of an advance directive, a life-saving blood transfusion can be given to a Jehovah's Witness who •
Block Time Remaining: 00:42:22
28
29 TIMEDTUTOR

03:45�
T• l/vl l/1 •
1M
1 •
=: ltem _25of30
:. ? 61 � � , � 0
2 Question Id: 3616 - ?Mark <J
Previous Next
[:> Tutorial Lab Values Notes Calculator Reverse Color Text Zoom
3 •
4 (Choice A) Case reports have shown some success in using high-dose erythropoietin, crystalloids, intravenous
5 iron, and other products to address massive hemorrhage wi thout a blood transfusion. These treatments could be
6
discussed if the patient was conscious and refused a blood transfusion.
7
8 (Choice B) Ethics committees provide advice if disagreements or ambiguity exist between health care providers
and a patient/patient' s family on decisions involving withholding or wi thdrawing life-sustaining treatments. This
� patient needs treatment quickly, with no time for discussion with the ethics committee.
11
12 (Choice C) Court approval is not needed for a physician to proceed with emergency life-saving treatment for an
13 unconscious or unresponsive patient.
14
15 Educational objective:
16 In the absence of an advance directive, a life-saving blood transfusion can be given to a Jehovah's Wi tness who
17 lacks decision-making capacity.
18
19
20 References
21
• The contemporary approach to the care of Jehovah's witnesses.
22
23 • Ethical principles--emergency medicine.
24
• Operating one-handed: emergency treatment of Jehovah's Witnesses.
26
Copyright© UWo�d. All rigllts reserved.
27 •
Block Time Remaining: 00:42:22
28
29 TIMEDTUTOR

03:45�
T• l/vl l/1 •
1M
1 •
=: ltem _26of30
:. ? 61 � � , � 0
2 Question Id: 3625 - ?Mark <J
Previous Next
[:> Tutorial Lab Values Notes Calculator Reverse Color Text Zoom
3 •
4 A 36-year-old man comes to the emergency department complaining of 2 days of fever, headache, and vomiting.
5
He has no other medical problems and takes no medications. He does not use tobacco, alcohol, or illicit drugs.
6
7
He lives with his wife and 2 school-aged children. The patient's temperature is 40 C (104 F), blood pressure is
8 100/60 mm Hg, pulse is 100/min, and respirations are 16/min. Physical examination shows petechiae and purpura
on his trunk and lower extremities. The patient is unable to bend his neck to touch his chest. Examination of
� cerebrospinal fluid is consistent with meningococcal meningitis. The patient is informed of his diagnosis. He is told
11
that management will consist of hospitalization, antibiotic treatment, and isolation precautions. However, he
12
refuses hospital admission and insists on being treated at home. The patient is alert and oriented and appears to
13
14 understand the risk he poses to himself and others by refusing hospitalization. He also declines requests to
15 involve his wife in the decision-making process. Which of the following is the most appropriate next step in
16 management?
17
18 O A. Consult the hospital ethics committee
19
20 O B. Hospitalize and isolate the patient against his wishes
21
22
O C. Inform the patient's wife and ask her to convince him to accept hospitalization
23 O D. Obtain a court order to proceed with treatment
24
25 O E. Respect the patient's decision and arrange for home antibiotic therapy
27

Block Time Remaining: 00:42:20


28
29 TIMEDTUTOR

03:45�
T• l/vl l/1 •
1M
1 •
=: ltem _26of30
:. ? 61 � � , � 0
2 Question Id: 3625 - ?Mark <JPrevious Next
[:> Tutorial Lab Values Notes Calculator Reverse Color Text Zoom
3 •
He has no other medical problems and takes no medications. He does not use tobacco, alcohol, or illicit drugs.
4
He lives with his wife and 2 school-aged children. The patient's temperature is 40 C (104 F), blood pressure is
5
6 100/60 mm Hg, pulse is 100/min, and respirations are 16/min. Physical examination shows petechiae and purpura
7 on his trunk and lower extremities. The patient is unable to bend his neck to touch his chest. Examination of
8 cerebrospinal fluid is consistent with meningococcal meningitis. The patient is informed of his diagnosis. He is told
that management will consist of hospitalization, antibiotic treatment, and isolation precautions. However, he
� refuses hospital admission and insists on being treated at home. The patient is alert and oriented and appears to
11
12 understand the risk he poses to himself and others by refusing hospitalization. He also declines requests to
13 involve his wife in the decision-making process. Which of the following is the most appropriate next step in
14 management?
15
16 X@ A. Consult the hospital ethics committee [5%]
17
18 � B. Hospital ize and isolate the patient against his wishes [58%]
19
C. Inform the patient's wife and ask her to convince him to accept hospital ization [3%]
20
21 D. Obtain a court order to proceed with treatment [9%]
22
23 E. Respect the patient's decision and arrange for home antibiotic therapy [23%>]
24
25

lncorr�
27 (1, , 58%
L!!!. Answered correctl II • •I
28
Block Time Remaining: 00:42:17
29 TIMEDTUTOR

03:45�
T• l/vl l/1 •
1M
1 •
=: ltem _26of30
:. ? 61 � � , � 0
2 Question Id: 3625 - ?Mark <J
Previous Next
[:> Tutorial Lab Values Notes Calculator Reverse Color Text Zoom
3 •
This patient presents with meningococcal meningitis, a highly contagious disease that can lead to devastating
4
5 complications and outbreaks in the community. Treatment involves isolation, intravenous antibiotics, and
6 supportive care in an intensive care setting to monitor disease progression. Physicians should first encourage the
7 patient to voluntarily comply with treatment by fully explaining the illness, the need for isolation, and the risk he
8
poses to himself and others. The patient should also be educated about the potential harm he could cause if
treated at home.

11 A patient who does not comply voluntarily needs to be hospitalized against his/her wishes. It is ethical for the
12
physician to isolate a noncompliant patient until the patient no longer poses a risk. This patient should be placed in
13
an isolation room with droplet precautions until all his contacts receive prophylaxis. His wife and children have
14
15 already been exposed to the disease and will need prophylaxis with antibiotics (eg, rifampin, ciprofloxacin). All
16 other exposed close contacts (including medical staff) will need chemoprophylaxis.
17
18
In general, adult patients with intact decision-making capacity have the right to refuse treatment, even if it would be
19 life-saving. However, protection of individual patient rights (patient autonomy) must be balanced with a
20 p hysician's duty to protect the health of the public by mandating hospitalization and isolation of patients with a
21 communicable disease.
22
23 (Choice A) The hospital ethics committee should be consulted when there is an ethical dilemma. It is clear that
24 this patient must be hospitalized to protect the health and welfare of others; the physician should proceed with
25 hosp italization immediately.

27 (Choice C) Notifying the patient's wife is necessary, but it is inappropriate to burden her with the responsibility of •
Block Time Remaining: 00:42:17
28
29 TIMEDTUTOR

03:45�
T• l/vl l/1 •
1M
1 •
=: ltem _26of30
:. Mark ? 61 � � , � 0
2 Question Id: 3625
- -- . - - -
-?
- - -- - -
<J
Previous
-
Next
[:>--
--- - - - -- - -- - -
Tutorial Lab Values
-- -- -- - ---
Notes
-
Calculator Reverse Color Text Zoom
3 •
4 p hysician's duty to protect the health of the public by mandating hospitalization and isolation of patients with a
5 communicable disease.
6
7
(Choice A) The hospital ethics committee should be consulted when there is an ethical dilemma. It is clear that
8 this patient must be hospitalized to protect the health and welfare of others; the physician should proceed with
hospitalization immediately.

11
(Choice C) Notifying the patient's wife is necessary, but it is inappropriate to burden her with the responsibility of
12 convincing him to accept hospitalization and treatment.
13
(Choice 0) A court order is not required in this situation. The medical community has already agreed that it is
14
15
imperative to isolate a patient with a communicable disease, even against his/her wishes.
16 (Choice E) This is one of the rare instances in which patient autonomy should be overridden due to the grave risk
17
posed to others.
18
19 Educational objective:
20
Patients have the right to refuse treatment except when doing so poses a serious threat to public health. In these
21
cases, the physician is justified in restricting individual liberties until the public's health is no longer at ri sk.
22
23
24 References
25
• Antibiotics for preventing meningococcal infections
27 .,, . . . . ... uarantine and isolation measures.
Block Time Remaining: 00:42:17
28
29 TIMEDTUTOR

03:45�
T• l/vl l/1 •
1M
1 •
=: ltem _27 of30
:. ? 61 � � , � 0
2 Question Id: 13533 - ?Mark <J
Previous Next
[:> Tutorial Lab Values Notes Calculator Reverse Color Text Zoom
3 •
4 A 34-year-old man is admitted to the hospital due to recurrent pancreatitis following a 2-week period of heavy
5 alcohol consumption. He has a long history of alcohol abuse and has been through inpatient alcohol treatment
6
programs on multiple occasions. The patient realizes he needs to stop, but says he drinks when he is home alone
7 i
8
and feels stressed. Medical history is notable for hypertension and type 2 diabetes mellitus, treated with lisinopr l
and metformin. Blood pressure is 146/85 mm Hg and pulse is 72/min. BMI is 46 kg/m2. On examination, the


patient appears comfortable. Fasting glucose is 183 mg/dL and hemoglobin A1c is 8.2%. The patient says he is
11 not willing to return to an inpatient alcohol treatment program after hospital discharge but might consider outpatient
12
counseling. Which of the following is the most appropriate response to this patient?
13
14
15
O A. I am concerned that you do not understand how alcohol affects your diabetes.
16 O B. I really think that going back to inpatient alcohol treatment is in your best interest.
17
18 O C. If you don 't take your alcoholism more seriously, one day it will kill you.
19
20
O D. Let's schedule an appointment with an outpatient program that can see you as soon as possible.
21 O E. Tell me some ways that outpatient alcohol treatment might help you.
22
23 O F. Whenever you are ready to be serious about your problems, I will be here to help you.
24
25
26 Submit

Block Time Remaining: 00:42:16
28
29 TIMEDTUTOR

03:45�
T• l/vl l/1 •
1M
1 •
=: ltem _27 of30
:. ? 61 � � , � 0
2 Question Id: 13533 - ?Mark <J
Previous Next
[:> Tutorial Lab Values Notes Calculator Reverse Color Text Zoom
3 •
4 A 34-year-old man is admitted to the hosp ital due to recurrent pancreatitis following a 2-week period of heavy
5
alcohol consumption. He has a long history of alcohol abuse and has been through inpatient alcohol treatment
6
7
programs on multiple occasions. The pati ent realizes he needs to stop, but says he drinks when he is home alone
8 and feels stressed. Medical history is notable for hypertension and type 2 diabetes mellitus, treated with lisinopril
and metformin. Blood pressure is 146/85 mm Hg and pulse is 72/min. BMI is 46 kg/m2. On examination, the
� patient appears comfortable. Fasting glucose is 183 mg/dL and hemoglobin A1c is 8.2%. The patient says he is
11
not willing to return to an inpatient alcohol treatment program after hospital discharge but might consider outpatient
12
13
counseling. Which of the following is the most appropr iate response to this patient?
14
15
A. I am concerned that you do not understand how alcohol affects your diabetes. [1%]
16
B. I really think that going back to inpatient alcohol treatment is in your best interest. [2%]
17
18 x,......,_
- C. If you don 't take your alcoholism more seriously, one day it will kill you. [0%]
19
20 D. Let's schedule an appointment with an outpatient program that can see you as soon as possible. [24%]
21
E. Tell me some ways that outpatient alcohol treatment might help you. [69%>]
22
23 F. Whenever you are ready to be serious about your problems, I will be here to help you. [1%>]
24
25
26
Incorrect
28
29
1M
1 •
=: ltem _27 of30
:. ? 61 � � , � 0
2 Question Id: 13533 - ?Mark <J Previous Next
[:> Tutorial Lab Values Notes Calculator Reverse Color Text Zoom
3 •
Stages of change model
4
5 Stage Motivational interviewing
6
7 • Encourage patient to evaluate
8 Not ready to c hange; patient does not consequences of current behavior
Precontemplation
acknowl edge negative consequences • Explain & personalize the risk
� • Recommending action is premature
11
12 Thinking of changing; patient • Encourage evaluation of pros & cons of
13 Contemplation acknowl edges consequences but is behavior change
14
ambivalent • Promote new, positive behaviors
15
16 • Encourage small initial steps
17 Ready to change; patient decides to
Preparation • Reinforce positive-outcome
18 change
19
expectations
20
• Help identify appropriate change
21
Making change; patient makes specific, strategies & enlist social support
22 Action
23 overt changes • Promote self-efficacy for dealing with
24 obstacles
25
26
• Follow-up support; reinforce intrinsic
Changes integrated into patient's life;
Maintenance rewards
focus on rela se revention
Block Time Remaining: 00:42:13
28
29 TIMEDTUTOR

03: 46 �
T• l/vl l/1 •
1M
1 •
=: ltem 27 of30
_ :. ? 61 � � , � 0
2 Question Id: 13533 - ? Mark <J
Previous Next
[:> Tutorial Lab Values Notes Calculator Reverse Color Text Zoom
3 •
Thinking of changing; patient • Encourage evaluation of pros & cons of
4
5 Contemplation acknowl edges consequences but is behavior change
6 ambivalent • Promote new, positive behaviors
7
8
• Encourage small initial steps
Ready to c hange; patient decides to
Preparation • Reinforce positive-outcome
change

expectations
11
12 • Help identify appropriate change
13 Making change; patient makes specific, strategies & enlist social support
Action
14 overt changes • Promote self-efficacy for dealing with
15
obstacles
16
17 • Follow-up support; reinforce intrinsic
18
Changes integrated into patient's life;
Maintenance rewards
19 focus on relapse prevention
• Develop relapse prevention strategies
20
21 Behavior is automatic; changes
22 Identification • Praise changes
incorporated into sense of self
23
24 The psychotherapeutic technique of motivational interviewing is used to encourage patients to change
25
maladaptive behaviors. Interventions are geared to the patient's stage of readiness to change. Elements include
26
a nonjudgmental stance focusing on the patient' s intrinsic motivation to change , acknowledging ambivalence and

Block Time Remaining: 00:42:13


28
29 TIMEDTUTOR

03: 46 �
T• l/vl l/1 •
1M
1 •
=: ltem _27 of30
:. ? 61 � � , � 0
2 Question Id: 13533 - ?Mark <J
Previous
[:> Next Tutorial Lab Values Notes Calculator Reverse Color Text Zoom
3 •
The psychotherapeutic technique of motivational interviewing is used to encourage patients to change
4
5 maladaptive behaviors. Interventions are geared to the patient's stage of readiness to change. Elements include
6 a nonjudgmental stance focusing on the patient' s intrinsic motivation to change, acknowl edging ambivalence and
7 resistance to change, and supporting the patient's autonomy and efforts to take responsibility.
8
This patient has recurrent alcoholic pancreatitis and has failed previous attempts at inpatient alcohol rehabilitation,


but he is willing to consider outpatient therapy (contemplation stage). Appropri ate steps at this point would be to
11 explain how referral for outpatient treatment might work, discuss the pros and cons of outpatient treatment
12
compared to inpatient treatment, and assess the patient's conviction (understanding of the importance of change)
13
and confidence (self-perceived ability to change). The physician should allow the patient time to consider all
14
15 options and should also arrange follow-up for planning the next steps in treatment.
16
(Choices A and C) A basic discussion of disease processes and complications can provide motivation for change
17
18
and is most appropriate for a patient in the precontemplation stage. However, this patient acknowledges that
19 change is needed and is willing to consider at least some treatment options. Continued emphasis on
20 consequences from lack of change is likely only to promote anxiety and frustration.
21
22
(Choices B and D) These statements do not acknowledge patient autonomy. Even if the physician is certain that
23 inpatient treatment would benefit the patient, a better understanding of the patient's perspective is needed before
24 pushing him to do what he does not want. Making arrangements to schedule an appointment with a treatment
25 program before the patient is agreeable may diminish his commitment to treatment. Such acti ons are more
26
appropriate for a patient in a preparation stage who can collaborate with the physician and take some responsibil ity

Block Time Remaining: 00:42:13


28
29 TIMEDTUTOR

03: 46 �
T• l/vl l/1 •
1M
1 •
=: ltem _27 of30
:. ? 61 � � , � 0
2 Question Id: 13533 - ?Mark <J
Previous Next
[:> Tutorial Lab Values Notes Calculator Reverse Color Text Zoom
3 •
and is most appropriate for a patient in the precontemplation stage. However, this patient acknowledges that
4
change is needed and is willing to consider at least some treatment options. Continued emphasis on
5
6 consequences from lack of change is likely only to promote anxiety and frustration.
7
(Choices B and D ) These statements do not acknowledge patient autonomy. Even if the physician is certain that
8
inpatient treatment would benefit the patient, a better understanding of the patient's perspective is needed before
pushing him to do what he does not want. Making arrangements to schedule an appointment with a treatment

11 program before the patient is agreeable may diminish his commitment to treatment. Such acti ons are more
12 appropriate for a patient in a preparation stage who can collaborate with the physician and take some responsibility
13
for treatment.
14
15 (Choice F) It is important to convey to all patients, in all stages of change, that their physician is committed to their
16 benefit and is willing to help. However, this patient's willingness to consider treatment options shows that he takes
17
his condition seriously, and this statement would likely be seen as condescending.
18
19 Educational objective:
20 Contemplation is a stage of behavior change in which a patient is willing to consider changes to maladaptive
21
behaviors but has not yet decided to pursue specific action. Patients considering behavioral change should be
22
23
assessed for conviction (understanding importance of change) and confidence (self-perceived ability to change).
24
25 References
26
• Diabetes self-management: facilitating lifestyle change.

Block Time Remaining: 00:42:13


28
29 TIMEDTUTOR

03: 46 �
T• l/vl l/1 •
1M
1 �
=: ltem _28of30
:. ? 61 � � , � 0
2 Question Id: 3624 - ?Mark <J
Previous Next
[:> Tutorial Lab Values Notes Calculator Reverse Color Text Zoom
3
4 A previously healthy 9-year-old boy is evaluated for a deep puncture wound to the foot. He was walking barefoot
5
on the beach when he stepped on a rusty nail. His parents cleaned the wound with soap and water and applied
6
7
pressure to it. The boy takes no medications. He has not received any vaccinations due to parental preference.
8 The boy's parents are divorced and have joint custody of him. The indications, benefits, and risks of tetanus
vaccination are discussed with both parents. The mother consents to the proposed tetanus vaccination and
� requests that it be administered immediately. However, his father refuses to agree to the vaccination as he prefers
11
to explore herbal remedies. Which of the following is the most appropriate next step in management of this
12
patient?
13
14
15
O A. Ask the patient about his wishes and proceed with his decision
16 O B. Consult the hospital's ethics committee
17
18 O C. Obtain a court order to proceed with the proposed treatment
19
20
O D. Proceed with the proposed treatment
21
22
0 E. Withhold the proposed treatment
23
24
Submit
25
26
27

Block Time Remaining: 00:42:12


29

03: 46 �
T• l/vl l/l •
1M
1 •
=: ltem _28of30
:. ? 61 � � , � 0
2 Question Id: 3624 - ?Mark <JPrevious Next
[:> Tutorial Lab Values Notes Calculator Reverse Color Text Zoom
3 •
4 A previously healthy 9-year-old boy is evaluated for a deep puncture wound to the foot. He was walking barefoot
5
on the beach when he stepped on a rusty nail. His parents cleaned the wound with soap and water and applied
6
7
pressure to it. The boy takes no medications. He has not received any vaccinations due to parental preference.
8 The boy's parents are divorced and have joint custody of him. The indications, benefits, and risks of tetanus
vaccination are discussed with both parents. The mother consents to the proposed tetanus vaccination and
� requests that it be administered immediately. However, his father refuses to agree to the vaccination as he prefers
11
to explore herbal remedies. Which of the following is the most appropriate next step in management of this
12
patient?
13
14
15
X@ A. Ask the patient about his wishes and proceed with his decision [1°/o]
16 8. Consult the hospital's ethics committee [9%]
17
18 C. Obtain a court order to proceed with the proposed treatment [6%]
19
20
D. Proceed with the proposed treatment [80%]
21
E. Wi thhold the proposed treatment [1 %]
22

I
23
24
I ncorrec�
25 (1, , 80% fl\ 4 Seconds i==I 10/07/2018
Correct answer L!!!. Answered correctly '.::; Time Spent l.!::.I Last Updated
26
D
27 •
Block Time Remaining: 00:42:09
29

03: 46 �
T• l/vl l/l •
1M
� � , � 0

1 ltem 28of30
=: _
Question Id: 3624
:.
?
Mark <J
Previous Next Lab Values
? 61
Notes Calculator Reverse Color Text Zoom
2 - [:> Tutorial
3 •
4
Circumstances in which minors can provide their own consent
5 • Emergency care (all states)
6
• Sexually transmitted infection (all states)
7
Medical emancipation • Mental health and substance abuse treatment
8
• Pregnancy care
• Contraception

11
12
• Financially independent
13 • Parent
14 Legal emancipation • Married
15 • Active military servi ce
16
• High school graduate
17
18 Note: The age of legal and medical emancipation varies by state.
19
20 This patient is an unemancipated minor, defined as an individual age <18 years who requires consent by a
21 parent or legal guardian for medical or surgical treatment. In all states, minors may receive acute care without
22
consent if obtaining it would delay treatment. The parents or legal guardians must be notified as soon as possible.
23
24
Many states also allow minors to consent without parental notification for contraception, treatment of sexually
25 transmitted diseases, prenatal care, mental health services, and substance abuse rehabilitation (Table).
26
According to the Ameri can Academy of Pediatrics Committee on Bioethics, a parental decision to decline routine
27
.. .. . - . . . - . . . - - . . . ·· -- - - - -- - .- . . . - . . . - . .. .. .
Block Time Remaining: 00:42:09
29

03: 46 �
T• l/vl l/l •
1M
1 �
=: ltem _28of30
:. ? 61 � � , � 0
Question Id: 3624 - ?Mark <JPrevious Next
[:> Tutorial Lab Values Notes Calculator Reverse Color Text Zoom
. . .. . .
2
3 .. g .. .. y : p
4 immunization for an unvaccinated child is often tolerated as the risk of acquiring infection in a community wi th herd
5
immunity is relatively low. In contrast to elective vaccination, this patient is at high risk of tetanus infection from
6
7 the deep puncture wound and administration of tetanus vaccination is urgent.
8
In patients with divorced parents, only parents with custody may give consent for medical care. Parents wi th joint
custody are each able to consent, but consent from only one parent is all that is necessary to proceed with

11
treating the minor, especially when the decision is clearl y in the child's best interests. In all cases, treatment is
12 ideally discussed with both parents to reach an agreement that will help the child and facilitate cooperation
13 between the treatment team and the parents.
14
15 (Choice A) Unemancipated minors do not have the legal authority to consent for procedures; consent must be
16 obtained from at least one parent.
17
(Choice B) It is appropriate to consult the hospital's ethics committee if an ethical dilemma is present. This is not
18
19 the case with this patient as tetanus vaccination is in his best interest and the mother has already provided
20 consent.
21
(Choice C) If both parents refuse to consent to life-saving treatment, the physician would need to seek court
22
23 approval before administering necessary but nonurgent treatment (eg, chemotherapy for acute lymphoblastic
24 leukemia in a stable patient).
25
26
(Choice E) Withholding the proposed treatment is inappropriate as the mother has already consented, which is
27 considered legally sufficient to proceed. Parents cannot deny their children life-saving treatment unless the •
Block Time Remaining: 00:42:09
29

03: 46 �
T• l/vl l/l •
1M
1 •
=: ltem _28of30
:. ? 61 � � , � 0
2 Question Id: 3624 - ?Mark <J
Previous Next
[:> Tutorial Lab Values Notes Calculator Reverse Color Text Zoom
3 •
between the treatment team and the parents.
4
5 (Choice A) Unemancipated minors do not have the legal authority to consent for procedures; consent must be
6 obtained from at least one parent.
7
8 (Choice B) It is appropriate to consult the hospital's ethics committee if an ethical dilemma is present. This is not
the case with this patient as tetanus vaccination is in his best interest and the mother has already provided
� consent.
11
12 (Choice C) If both parents refuse to consent to life-saving treatment, the physician would need to seek court
13 approval before administering necessary but nonurgent treatment (eg, chemotherapy for acute lymphoblastic
14 leukemia in a stable patient).
15
16 (Choice E) Withholding the proposed treatment is inappropriate as the mother has already consented, which is
17 considered legally sufficient to proceed. Parents cannot deny their children life-saving treatment unless the
18
benefits are minimal or would not alter the prognosis.
19
20 Educational objective:
21 When caring for an unemancipated minor, informed consent from one parent or guardian is considered legally
22
sufficient to justify proceeding with therapy. Physicians should also provide care in urgent situations without
23
waiting for parental consent.
24
25
26 References
27

Block Time Remaining: 00:42:09


29

03: 46 �
T• l/vl l/l •
1M
1 �
=: ltem _29 of30
:. ? 61 � � , � 0
2 Question Id: 3614 - ?Mark <J
Previous Next
[:> Tutorial Lab Values Notes Calculator Reverse Color Text Zoom
3
4 A 36-year-old man has been hospitalized for the past 3 days for alcoholic hepatitis. His condition is slowly
5
improving and his prognosis is favorable. However, this is his third absence from his job in the past 6 months due
6
7
to health-related issues. The patient notified his employer that he is presently hospitalized and was informed that
8 he needs to provide the employer with proof of hosp italization or he could lose his job. In accordance with hospital
policy, the patient provides verbal consent for the physician to speak wi th his employer and disclose his health
� status. The employer proceeds to contact the physician directly by phone, requesting an update on the patient and
11
his likelihood of returning to work. Which of the following is the most appropriate response by the physician to the
12
employer?
13
14
15
O A. "I can confirm that the patient is currently hospitalized, and I anticipate he will be able to return to work."
16 O B. "I can disclose information to you only in the presence of the patient, so we need to schedule a
17
meeting."
18
19 O C. "I cannot discuss a patient's care with anyone without a written signed release of information."
20
21
O D. "I cannot verify your identity and therefore cannot disclose any information about the patient."
22 O E. "The diagnosis is alcoholic hepatitis and I expect him to recover and return to work in the coming days."
23
24
25
Submit
26
27

Block Time Remaining: 00:42:08

03: 46 �
T• l/vl l/l •
1M
1 •
=: ltem _29 of30
:. ? 61 � � , � 0
2 Question Id: 3614 - ?Mark <J
Previous Next
[:> Tutorial Lab Values Notes Calculator Reverse Color Text Zoom
3 •
4 A 36-year-old man has been hospitalized for the past 3 days for alcoholic hepatitis. His condition is slowly
5
improving and his prognosis is favorable. However, this is his third absence from his job in the past 6 months due
6
7
to health-related issues. The patient notified his employer that he is presently hospitalized and was informed that
8 he needs to provide the employer with proof of hosp italization or he could lose his job. In accordance with hospital
policy, the patient provides verbal consent for the physician to speak wi th his employer and disclose his health
� status. The employer proceeds to contact the physician directly by phone, requesting an update on the patient and
11
his likelihood of returning to work. Which of the following is the most appropriate response by the physician to the
12
employer?
13
14
_., A. "I can confirm that the patient is currently hospitalized, and I anticipate he will be able to return to work."
15
16 [71%]
17 X � B. "I can disclose information to you only in the presence of the patient, so we need to schedule a
18
meeting." [2%]
19
20 C. "I cannot discuss a patient's care with anyone without a written signed release of information." [15%]
21
D. "I cannot verify your identity and therefore cannot disclose any information about the patient." [9%]
22
23 E. "The diagnosis is alcoholic hepatitis and I expect him to recover and return to work in the coming days."
24
[2%]
25
26

Block Time Remaining: 00:42:05

03: 46 �
T• l/vl l/l •
1M
1 •
=: ltem _29 of30
:. ? 61 � � , � 0
2 Question Id: 3614 - ?Mark <J
Previous Next
[:> Tutorial Lab Values Notes Calculator Reverse Color Text Zoom
3 •
The Health Insurance Portability and Accountability Act (HIPAA) privacy rule outlines the handling of protected
4
health information. The main goal is to protect patients' privacy and confidentiality. A physician can respond to an
5
6 employer's request for health information only if the patient has specifically provided verbal or written
7 authorization for release of information to the employer. Because this patient has given his physician verbal
8 permission to speak with his employer and discuss his care, in accordance with hospital policy, the physician can
provide this information. In general, a written (rather than verbal) release of information document is preferred to
� protect both the patient and physician from any misunderstanding, but this is not a HI PAA requirement (Choice C).
11
12 (Choice B) If prior authorization for disclosure has been given, the patient's presence is not required.
13
14 (Choice 0) HIPAA does not require the provider to obtain proof of identity of phone callers before speaking with
15 them. However, in some cases, a health care provider or hospital may establish additional rules for verification of a
16 phone caller's identity.
17
18 (Choice E) A central aspect of the HI PAA privacy rule is the principle of "minimum necessary" disclosure. The
19 physician should disclose only the minimum amount of protected health information needed to respond to the
20 spec ific request. In this case, the physician is asked to confirm that the patient is hospital ized and comment on his
21
likelihood of returning to work. The physician does not need to disclose the patient's diagnosis to the employer and
22
23
should avoid doing so as this sensitive information could jeopardize the patient's future employment.
24 Educational objective:
25
The Health Insurance Portability and Accountability Act protects health information by requiring verbal or written
26
27
authorization for release of information. Hospitals and physicians' offices frequently have additional policies

Block Time Remaining: 00:42:05

03: 46 �
T• l/vl l/l •
1M
1 •
=: ltem _29 of30
:. ? 61 � � , � 0
2 Question Id: 3614 - ?Mark <J
Previous Next
[:> Tutorial Lab Values Notes Calculator Reverse Color Text Zoom
3 •
permission to speak with his employer and discuss his care, in accordance with hospital policy, the physician can
4
5 provide this information. In general, a written (rather than verbal) release of information document is preferred to
6 protect both the patient and physician from any misunderstanding, but this is not a HI PAA requirement (Choice C).
7
8
(Choice B) If prior authorization for disclosure has been given, the patient's presence is not required.

(Choice D) HIPAA does not require the provider to obtain proof of identity of phone callers before speaking with
� them. However, in some cases, a health care provider or hospital may establish additional rules for verification of a
11
phone caller's identity.
12
13 (Choice E) A central aspect of the HI PAA privacy rule is the principle of "minimum necessary" disclosure. The
14
physician should disclose only the minimum amount of protected health information needed to respond to the
15
16
spec ific request. In this case, the physician is asked to confirm that the patient is hospital ized and comment on his
17 likelihood of returning to work. The physician does not need to disclose the patient's diagnosis to the employer and
18 should avoid doing so as this sensitive information could jeopardize the patient's future employment.
19
20
Educational objective:
21 The Health Insurance Portability and Accountability Act protects health information by requiring verbal or written
22 authorization for release of information. Hospitals and physicians' offices frequently have additional policies
23 requiring written forms for release of information and procedures to verify the identity of phone callers. It is
24
important that health care providers be familiar with these rules and disclose only the minimum necessary
25
26 information.
27 Copyright© UWo�d. All rigllts resenred. •
Block Time Remaining: 00:42:05

03: 47 �
T• l/vl l/l •
1M
1 •
=: ltem _30of30
:. ? 61 � � , � 0
2 Question Id: 4066 - ?Mark <J
Previous Next
[:> Tutorial Lab Values Notes Calculator Reverse Color Text Zoom
3 •
4 A medical resident on call is asked to see a 72-year-old woman who has a headache. The medical record
5
indicates that she was admitted 5 days ago after a fall that caused a hip fracture requiring surgical correction. The
6
7
patient developed postoperative chest pain and was transferred to the medical service for further care. She is
8 stabilized and receiving morphine for her pain but frequently calls nurses to say that she is uncomfortable.
According to the nurse, the patient developed an intermittent headache earlier in the day that recurred an hour ago
� after her daughter called to say she would be unable to visit today. When she sees the resident, the patient snaps,
11
"You look so young. I hope you know what you're doing." The resident asks a few questions about her headache,
12
but she gives very little information. When the resident attempts to perform a basic physical examination, the
13
14 patient yells, "Don't touch me. I'm going to complain to the hospital administrators about you because you have no
15 idea how to properl y deal with a patient." Which of the following is the most appropriate response?
16
17 O A. "As you may know, this is a teaching hospital; I can assure you that I am well trained and can consult an
18 attending physician if necessary."
O
19
20 B. "I am sorry you are unhappy with your care; you are free to file a complaint as described in the patient
21 bill of rights."
22
23
O C. "I see that you're upset; I imagine that you might be disappointed that your daughter was unable to
visit."
24
25 O D. "I see that you're upset; would you prefer to be seen by my supervising attending physician?"

O
26
27
E. "I understand that you are upset, but I can best help you if you allow me to proceed with the

Block Time Remaining: 00:42:00


28
29 TIMEDTUTOR

03: 47 �
T• l/vl l/1 •
1M
1 •
=: ltem _30of30
:. ? 61 � � , � 0
2 Question Id: 4066 - ?Mark <J
Previous Next
[:> Tutorial Lab Values Notes Calculator Reverse Color Text Zoom
3 stabilized and receiving morphine for her pain but frequently calls nurses to say that she is uncomfortable. •
4
According to the nurse, the patient developed an intermittent headache earlier in the day that recurred an hour ago
5
after her daughter called to say she would be unable to visit today. When she sees the resident, the patient snaps,
6
7 "You look so young. I hope you know what you're doing." The resident asks a few questions about her headache,
8 but she gives very little information. When the resident attempts to perform a basic physical examination, the
patient yells, "Don't touch me. I'm going to complain to the hospital administrators about you because you have no
� idea how to properl y deal with a patient." Which of the following is the most appropriate response?
11
12
13
O A. "As you may know, this is a teaching hospital; I can assure you that I am well trained and can consult an
attending physician if necessary. "
14
15 O B. "I am sorry you are unhappy with your care; you are free to file a complaint as described in the patient
16 bill of rights."
O C. "I see that you're upset; I imagine that you might be disappointed that your daughter was unable to
17
18
19 visit."
20
21
O D. "I see that you're upset; would you prefer to be seen by my supervising attending physician?"

22 O E. "I understand that you are upset, but I can best help you if you allow me to proceed with the
23 examination and make a diagnosis."
O
24
25 F. "May I ask what is upsetting you so that I can help you as best I can?"
26
27

Block Time Remaining: 00:41 :57


28
29 TIMEDTUTOR

03: 47 �
T• l/vl l/1 •
1M
1 •
=: ltem _30of30
:. ? 61 � � , � 0
2 Question Id: 4066 - ?Mark <JPrevious Next
[:> Tutorial Lab Values Notes Calculator Reverse Color Text Zoom
3 •
4 after her daughter called to say she would be unable to visit today. When she sees the resident, the patient snaps,
5 "You look so young. I hope you know what you're doing." The resident asks a few questions about her headache,
6 but she gives very little information. When the resident attempts to perform a basic physical examination, the
7
patient yells, "Don't touch me. I'm going to complain to the hospital administrators about you because you have no
8
idea how to properl y deal with a patient." Which of the following is the most appropriate response?

� A. "As you may know, this is a teaching hospital; I can assure you that I am well trained and can consult an
11
12
attending physician if necessary. " [3%]
13 B. "I am sorry you are unhappy with your care; you are free to file a complaint as described in the patient
14
bill of rights." [0%]
15
16 C. "I see that you're upset; I imagine that you might be disappointed that your daughter was unable to
17 visit." [2%]
18
D. "I see that you're upset; would you prefer to be seen by my supervising attending physician?" [6%]
19
20 X (" E. "I understand that you are upset, but I can best help you if you allow me to proceed with the
21
examination and make a diagnosis." [13%>]
22
23 F. "May I ask what is upsetting you so that I can help you as best I can?" [73%]
24
25
26
lncorreq
27 (1, , 73% fl\ 11 Seconds i::==I 09/22/2018
Correct answer L!!!. Answered correctly "-.::.; TimeSpent l.!!!.I Last Updated •
Block Time Remaining: 00:41 :54
28
29 TIMEDTUTOR

03: 47 �
T• l/vl l/1 •
1M
1 •
=: ltem _30of30
:. ? 61 � � , � 0
2 Question Id: 4066 - ?Mark <J
Previous Next
[:> Tutorial Lab Values Notes Calculator Reverse Color Text Zoom
3 •
This patient is irritable, angry, and verbally abusive to the resident physician. Although the immediate cause of her
4
anger is unclear, her complicated medical course, general discomfort and pain, and probable disappointment that
5
6 her daughter did not visit are likely contributing to her distress. Although the situation is challenging, it is the
7 physician's responsibility to be nondefensive, defuse the patient's anger, and attempt to preserve the physician­
8 patient relationship. Acknowledging the patient's distress, remaining nondefensive, and asking open-ended
questions regarding what is upsetting the patient are the best means of establishing an open dialogue.
� Professionalism should be maintained at all times, and the physician should not retaliate with defensive or negative
11
12 comments, which will only worsen the situation.
13
(Choice A) Reminding the patient that she is in a teaching hospital and mentioning a lack of experience will likely
14
further anger her and provide her with additional opportunities to criticize the resident.
15
16 (Choice B) Although patients have the right to file a complaint if dissatisfied with their care, suggesting this
17
approach in a situation where there is no evidence of poor treatment is inappropriate and does nothing to resolve
18
the issue or improve the physician-patient relationship.
19
20 (Choice C) Although the patient's disappointment that her daughter did not visit may be a contributing factor, it is
21
inappropriate to assume this is the reason for her distress; this premature interpretation is likely to further
22
23
aggravate the patient.
24 (Choice D) The best initial approach is to acknowledge the patient's distress and ask an open-ended question. It
25
is likely that this situation can be resolved by the resident without involving the attending physician.
26
27 (Choice E) Although this patient's cooperation is necessary to complete the physical examination and make a •
Block Time Remaining: 00:41 :54
28
29 TIMEDTUTOR

03: 47 �
T• l/vl l/1 •
1M
1 •
=: ltem _30of30
:. ? 61 � � , � 0
2 Question Id: 4066 - ?Mark <J
Previous Next
[:> Tutorial Lab Values Notes Calculator Reverse Color Text Zoom
3 •
4 (Choice A) Reminding the patient that she is in a teaching hospital and mentioning a lack of experience will likely
5 further anger her and provide her with additional opportunities to criticize the resident.
6
7 (Choice B) Although patients have the right to file a complaint if dissatisfied with their care, suggesting this
8 approach in a situation where there is no evidence of poor treatment is inappropriate and does nothing to resolve
the issue or improve the physician-patient relationship.

11
(Choice C) Although the patient's disappointment that her daughter did not visit may be a contributing factor, it is
12 inappropriate to assume this is the reason for her distress; this premature interpretation is likely to further
13 aggravate the patient.
14
15 (Choice 0) The best initial approach is to acknowledge the patient's distress and ask an open-ended question. It
16 is likely that this situation can be resolved by the resident without involving the attending physician.
17
(Choice E) Although this patient's cooperation is necessary to complete the physical examination and make a
18
19 diagnosis, her anger should be addressed first to facilitate her cooperation.
20
Educational objective:
21
22
When dealing with an angry patient, the most appropriate response is to remain nondefensive, acknowledge that
23 the patient is upset, and begin the discussion with an open-ended question.
24
25
References
26
27 • Managing difficult encounters: understanding physician, patient, and situational factors. •
Block Time Remaining: 00:41 :54
28
29 TIMEDTUTOR

03: 47 �
T• l/vl l/1 •

You might also like